You are on page 1of 78

DOCTRINE:

Bail acts as a reconciling mechanism to accommodate both the accused’s interest in pretrial
liberty and society’s interest in assuring the accused’s presence at trial. Bail, the security given
by an accused who is in the custody of the law for his release to guarantee his appearance
before any court as may be required, is the answer of the criminal justice system to a vexing
question: what is to be done with the accused, whose guilt has not yet been proven, in the
“dubious interval,” often years long, between arrest and final adjudication?

FACTS:
Charged with the murder of Rafael de las Alas, petitioner Jose Antonio Leviste was convicted by
the Regional Trial Court of Makati City for the lesser crime of homicide and sentenced to suffer
an indeterminate penalty of six years and one day of prision mayor as minimum to 12 years and
one day of reclusion temporal as maximum. He appealed his conviction to the Court of Appeals.
Pending appeal, he filed an urgent application for admission to bail pending appeal, citing his
advanced age and health condition, and claiming the absence of any risk or possibility of flight
on his part.
The Court of Appeals denied petitioner’s application for bail. It invoked the bedrock principle in
the matter of bail pending appeal, that the discretion to extend bail during the course of appeal
should be exercised “with grave caution and only for strong reasons.” Citing well-established
jurisprudence, it ruled that bail is not a sick pass for an ailing or aged detainee or a prisoner
needing medical care outside the prison facility.

For purposes of determining whether petitioner’s application for bail could be allowed pending
appeal, the Court of Appeals also considered the fact of petitioner’s conviction. It made a
preliminary evaluation of petitioner’s case and made a prima facie determination that there was
no reason substantial enough to overturn the evidence of petitioner’s guilt. Petitioner’s motion
for reconsideration was denied.

ISSUE:
Whether or not the discretionary nature of the grant of bail pending appeal mean that bail should
automatically be granted absent any of the circumstances mentioned in the third paragraph of
Section 5, Rule 114 of the Rules of Court?

RULING:

Section 5, Rule 114 of the Rules of Court provides:


“Sec. 5. Bail, when discretionary.—Upon conviction by the Regional Trial Court of an
offense not punishable by death, reclusion perpetua, or life imprisonment, admission to
bail is discretionary. The application for bail may be filed and acted upon by the trial court
despite the filing of a notice of appeal, provided it has not transmitted the original record to the
appellate court. However, if the decision of the trial court convicting the accused changed the
nature of the offense from non-bailable to bailable, the application for bail can only be filed with
and resolved by the appellate court.
Should the court grant the application, the accused may be allowed to continue on
provisional liberty during the pendency of the appeal under the same bail subject to the consent
of the bondsman.
If the penalty imposed by the trial court is imprisonment exceeding six (6) years, the
accused shall be denied bail, or his bail shall be cancelled upon a showing by the
prosecution, with notice to the accused, of the following or other similar circumstances:
(a) That he is a recidivist, quasi-recidivist, or habitual delinquent, or has
committed the crime aggravated by the circumstance of reiteration;
(b) That he has previously escaped from legal confinement, evaded sentence,
or violated the conditions of his bail without a valid justification;
(c) That he committed the offense while under probation, parole, or conditional
pardon;
(d) That the circumstances of his case indicate the probability of flight if
released on bail; or
(e) That there is undue risk that he may commit another crime during the
pendency of the appeal.

The appellate court may, motu proprio or on motion of any party, review the resolution of the
Regional Trial Court after notice to the adverse party in either case.” 

The third paragraph of Section 5, Rule 114 applies to two scenarios where the penalty imposed
on the appellant applying for bail is imprisonment exceeding six years. The first scenario deals
with the circumstances enumerated in the said paragraph (namely, recidivism, quasi-recidivism,
habitual delinquency or commission of the crime aggravated by the circumstance of reiteration;
previous escape from legal confinement, evasion of sentence or violation of the conditions of his
bail without a valid justification; commission of the offense while under probation, parole or
conditional pardon; circumstances indicating the probability of flight if released on bail; undue
risk of committing another crime during the pendency of the appeal; or other similar
circumstances) not present. The second scenario contemplates the existence of at least one of
the said circumstances.

In the first situation, bail is a matter of sound judicial discretion. This means that, if none of the
circumstances mentioned in the third paragraph of Section 5, Rule 114 is present, the appellate
court has the discretion to grant or deny bail. An application for bail pending appeal may be
denied even if the bail-negating26 circumstances in the third paragraph of Section 5, Rule 114
are absent. In other words, the appellate court’s denial of bail pending appeal where none of the
said circumstances exists does not, by and of itself, constitute abuse of discretion.

On the other hand, in the second situation, the appellate court exercises a more stringent
discretion, that is, to carefully ascertain whether any of the enumerated circumstances in fact
exists. If it so determines, it has no other option except to deny or revoke bail pending appeal.
Conversely, if the appellate court grants bail pending appeal, grave abuse of discretion will
thereby be committed.
Given these two distinct scenarios, therefore, any application for bail pending appeal should
be viewed from the perspective of two stages: (1) the determination of discretion stage, where
the appellate court must determine whether any of the circumstances in the third paragraph of
Section 5, Rule 114 is present; this will establish whether or not the appellate court will exercise
sound discretion or stringent discretion in resolving the application for bail pending appeal and
(2) the exercise of discretion stage where, assuming the appellant’s case falls within the first
scenario allowing the exercise of sound discretion, the appellate court may consider all relevant
circumstances, other than those mentioned in the third paragraph of Section 5, Rule 114,
including the demands of equity and justice; on the basis thereof, it may either allow or disallow
bail.
On the other hand, if the appellant’s case falls within the second scenario, the appellate
court’s stringent discretion requires that the exercise thereof be primarily focused on the
determination of the proof of the presence of any of the circumstances that are prejudicial to the
allowance of bail. This is so because the existence of any of those circumstances is by itself
sufficient to deny or revoke bail. Nonetheless, a finding that none of the said circumstances
is present will not automatically result in the grant of bail. Such finding will simply
authorize the court to use the less stringent sound discretion approach.
Petitioner disregards the fine yet substantial distinction between the two different situations
that are governed by the third paragraph of Section 5, Rule 114. Instead, petitioner insists on a
simplistic treatment that unduly dilutes the import of the said provision and trivializes the
established policy governing the grant of bail pending appeal.

In particular, a careful reading of petitioner’s arguments reveals that it interprets the third
paragraph of Section 5, Rule 114 to cover all situations where the penalty imposed by the trial
court on the appellant is imprisonment exceeding six years. For petitioner, in such a situation,
the grant of bail pending appeal is always subject to limited discretion, that is, one restricted to
the determination of whether any of the five bail-negating circumstances exists. The
implication of this position is that, if any such circumstance is present, then bail will be denied.
Otherwise, bail will be granted pending appeal.

Petitioner’s theory therefore reduces the appellate court into a mere fact-finding body whose
authority is limited to determining whether any of the five circumstances mentioned in the third
paragraph of Section 5, Rule 114 exists. This unduly constricts its “discretion” into merely filling
out the checklist of circumstances in the third paragraph of Section 5, Rule 114 in all instances
where the penalty imposed by the Regional Trial Court on the appellant is imprisonment
exceeding six years. In short, petitioner’s interpretation severely curbs the discretion of the
appellate court by requiring it to determine a singular factual issue—whether any of the five bail-
negating circumstances is present.

Finally, laws and rules should not be interpreted in such a way that leads to unreasonable or
senseless consequences. An absurd situation will result from adopting petitioner’s interpretation
that, where the penalty imposed by the trial court is imprisonment exceeding six years, bail
ought to be granted if none of the listed bail-negating circumstances exists. Allowance of bail
pending appeal in cases where the penalty imposed is more than six years of imprisonment will
be more lenient than in cases where the penalty imposed does not exceed six years. While
denial or revocation of bail in cases where the penalty imposed is more than six years’
imprisonment must be made only if any of the five bail-negating conditions is present, bail
pending appeal in cases where the penalty imposed does not exceed six years imprisonment
may be denied even without those conditions.
CHUA VS CA
2007, SANDOVAL –GUTIERREZ, J:
DOCTRINE: After a judgment of conviction has been rendered by the trial court and
cancellation of the bail bond of the accused, his appropriate remedy against the court’s order
cancelling his bond is by filing with the Court of Appeals a motion to review the said order in the
same regular appeal proceedings, as an incident of his appeal—the filing of a separate petition
via a special civil action or special proceeding questioning such adverse order before the
appellate court is proscribed.
FACTS: Rufina Chua invest her money in stocks and made Chiok as the stockbroker. She
entrusted to him the amount of ₱9,563,900.00 for the purpose of buying shares of stocks in
bulk. Chiok spent the money. For payment, he issued two checks as payment but the checks
were dishonored for insufficient funds. She then charged Chiok of estafa.
Chiok failed to appear at the date of promulgation of judgment twice. The RTC issued an
Omnibus Order (a) denying respondent’s motion for reconsideration of the judgment of
conviction; (b) canceling his bail; and (c) giving him five (5) days from notice within which to
appear before the trial court, otherwise he would be arrested. Chiok appealed to CA filed
petition for certiorari with application for a temporary restraining order (TRO) and a writ
preliminary injunction assailing the trial court’s Omnibus Order canceling his bail. Thereafter,
trial court issued a warrant of arrest against respondent for his failure to appear despite the
lapse of the 5-day period provided in Omnibus Order. The CA issued a writ of preliminary
injunction enjoining the arrest of respondent.
ISSUE: Is Court of Appeals correct in the issuance of writ of preliminary injunction enjoining the
arrest of respondent?
HELD: NO. Firstly, the petition for certiorari with prayer for a TRO and a writ of preliminary is not
the proper recourse in assailing the trial court’s Omnibus Order canceling his bail. Section 5,
Rule 114 of the Revised Rules of Criminal Procedure provides:
SEC. 5. Bail, when discretionary. – Upon conviction by the Regional Trial Court of an offense
not punishable by death, reclusion perpetua or life imprisonment, admission to bail is
discretionary. The application for bail may be filed and acted upon by the trial court despite the
filing of a notice of appeal, provided it has not transmitted the original record to the appellate
court. However, if the decision of the trial court convicting the accused changed the nature of
the offense from non-bailable to bailable, the application for bail can only be filed with and
resolved by the appellate court, Should the court grant the application, the accused may be
allowed to continue on provisional liberty during the pendency of the appeal under the same bail
subject to the consent of the bondsman.
If the penalty imposed by the trial court is imprisonment exceeding six (6) years, the accused
shall be denied bail, or his bail shall be cancelled upon a showing by the prosecution, with
notice to the accused, of the following or other similar circumstances:
(a) That he is a recidivist, quasi-recidivist, or habitual delinquent, or has committed the crime
aggravated by the circumstance of reiteration;
(b) That he has previously escaped from legal confinement, evaded sentence, or violated the
conditions of his bail without valid justification;
(c) That he committed the offense while under probation, parole, or conditional pardon;
(d) That the circumstances of his case indicate the probability of flight if released on bail; or
(e) That there is undue risk that he may commit another crime during the pendency of the
appeal.1awphi1.nét
The appellate court may, motu proprio or ON MOTION OF ANY PARTY, review the resolution
of the Regional Trial Court after notice to the adverse party in either case.
It is clear from the last paragraph of the above provision that private respondent’s appropriate
remedy against the trial court’s May 28, 1999 Omnibus Order canceling his bail is by filing with
the Court of Appeals a motion to review the said order in the same regular appeal proceedings
in CA-G.R. CR No. 23309 he himself initiated. Such motion is an incident in his appeal. The
filing of a separate petition via a special civil action or special proceeding questioning such
adverse order before the appellate court is proscribed.
Such independent special civil action obviously contravenes the rule against multiplicity of suits
and constitutes forum shopping.
Hence, the Court of Appeals erred in not dismissing outright respondent’s petition for certiorari.
The basic rule is that such petition may only be availed of when "there is no appeal or any plain,
speedy and adequate remedy in the ordinary course of law."
Secondly, to be entitled to an injunctive writ, the applicant must show that (1) he has a clear
existing right to be protected; and (2) the acts against which the injunction is to be directed are
in violation of such right.
In this case, both requisites are absent. Respondent has no right to be freed on bail pending his
appeal from the trial court’s judgment. His conviction carries a penalty of imprisonment
exceeding 6 years (to be exact, 12 years of prision mayor, as minimum, to 20 years of reclusion
temporal, as maximum) which justifies the cancellation of his bail pursuant to the third
paragraph of Section 5 (b), (d) and (e) of Rule 114. Moreover, he failed to appear despite notice
during the promulgation of judgment. His inexcusable non-appearance not only violated the
condition of his bail that he "shall appear" before the court "whenever required" by the latter or
the Rules, but also showed the probability that he might flee or commit another crime while
released on bail.
At this point, we stress that when respondent did not appear during the promulgation of
judgment on January 26, 1999 despite notice, and without offering any justification therefor, the
trial court should have immediately promulgated its Decision.
If the judgment is for conviction and the failure of the accused to appear was without justifiable
cause, he shall lose the remedies available in these Rules against the judgment and the court
shall order his arrest. Within fifteen (15) days from promulgation of judgment, however, the
accused may surrender and file a motion for leave of court to avail of these remedies. He shall
state the reasons for his absence at the scheduled promulgation, and if he proves that his
absence was for a justifiable cause, he shall be allowed to avail of said remedies within fifteen
(15) days from notice.
Since respondent has not shown any right to be protected, the second requisite for the issuance
of a writ of preliminary injunction is obviously absent.
G.R. No. 129670.
February 1, 2000
MANOLET O. LAVIDES, Petitioner, vs. HONORABLE COURT OF APPEALS;

DOCTRINE: Art. III, §14(2) of the Constitution authorizing trials in absentia allows the accused
to be absent at the trial but not at certain stages of the proceedings, to wit: (a) at arraignment
and plea, whether of innocence or of guilt,9 (b) during trial whenever necessary for identification
purposes,and (c) at the promulgation of sentence, unless it is for a light offense, in which case
the accused may appear by counsel or representative. At such stages of the proceedings, his
presence is required and cannot be waived

Facts : Petitioner Manolet Lavides was arrested on April 3, 1997 for child abuse under R.A. No.
7610 (an act providing for stronger deterrence and special protection against child abuse,
exploitation and discrimination, providing penalties for its violation, and other purposes). Based
on the sworn statement of complainant and the affidavits of the arresting officers, which were
submitted at the inquest, an information for violation of Art. III, 5(b) of R.A. No. 7610 was filed on
April 7, 1997 against petitioner in the Regional Trial Court, Quezon City, where it was docketed
as Criminal Case No. Q-97-70550. Petitioner filed an "Omnibus Motion that the accused be
Allowed to Bail as a Matter of Right. On May 16, 1997, the trial court issued an order resolving
petitioners Omnibus Motion that the approval of the bail bonds shall be made only after the
arraignment to enable this Court to immediately acquire jurisdiction over the accused.The
accused was arraigned and post a bail.Petitioner takes issue with the Court of Appeals with
respect to its treatment of condition of the May 16, 1997 order of the trial court which makes
petitioner's arraignment a prerequisite to the approval of his bail bonds. His contention is that
this condition is void and that his arraignment was also invalid because it was held pursuant to
such invalid condition.

Issue: whether or not prerequisite of arraignment before admission to bail is an invalid


condition?

Held:
Yes
To condition the grant of bail to an accused on his arraignment would be to place him in a
position where he has to choose between (1) filing a motion to quash and thus delay his release
on bail because until his motion to quash can be resolved, his arraignment cannot be held, and
(2) foregoing the filing of a motion to quash so that he can be arraigned at once and thereafter
be released on bail. These scenarios certainly undermine the accused's constitutional right not
to be put on trial except upon valid complaint or information sufficient to charge him with a crime
and his right to bail. Although this condition is invalid, it does not follow that the arraignment of
petitioner on was also invalid. Contrary to petitioner's contention, the arraignment did not
emanate from the invalid condition that "approval of the bail bonds shall be made only after the
arraignment." Even without such a condition, the arraignment of petitioner could not be omitted.
In sum, although the condition for the grant of bail to petitioner is invalid, his arraignment and
the subsequent proceedings against him are valid.
People vs. Valdez 2015
Doctrine:
For purposes of determining whether a person can be admitted to bail as a matter of
right, it is the imposable penalty prescribed by law for the crime charged which should be
considered and, not the penalty to be actually imposed.
Facts:
The case stemmed from the Joint Affidavit executed by Sheila S. Velmonte-Portal and
Mylene T. Romero, both State Auditors of the Commission on Audit Region VI in Iloilo, who
conducted a post-audit of the disbursement vouchers of the Bacolod City Government.
Among the subjects thereof were the reimbursements of expenses of private respondent
Luzviminda S. Valdez (Valdez), a former mayor of Bacolod City, particularly: Based on the
verification conducted in the establishments that issued the official receipts, it was alleged that
the cash slips were altered/falsified to enable Valdez to claim/receive reimbursement from the
Government the total amount of P279,150.00 instead of only P4,843.25; thus, an aggregate
overclaim of P274,306.75.
The Public Assistance and Corruption Prevention Office (PACPO), Office of the
Ombudsman - Visayas received the joint affidavit, which was thereafter resolved adverse to
Valdez.
Consequently, Valdez was charged with eight cases four of which (SB-14-CRM-0317 to
0320) were for Violation of Section 3 (e) of Republic Act No. 3019, while the remaining half (SB-
14-CRM-0321 to 0324) were for the complex crime of Malversation of Public Funds thru
Falsification... of Official/Public Documents under Articles 217[5] and 171,[6] in relation to Article
48[7] of the Revised Penal Code (RPC). All the cases were raffled before public respondent.
Since the Ombudsman recommended "no bail" in SB-14-CRM-0321, 0322, and 0324,
Valdez, who is still at-large, caused the filing of a Motion to Set Aside No Bail Recommendation
and to Fix the Amount of Bail.
She argued that the three cases are bailable as a... matter of right because no
aggravating or modifying circumstance was alleged; the maximum of the indeterminate
sentence shall be taken from the medium period that ranged from 18 years, 8 months and 1 day
to 20 years; and applying Article 48 of the RPC, the imposable penalty is 20 years, which is the
maximum of the medium period.
Petitioner countered in its Comment/Opposition that the Indeterminate Sentence Law
(ISL) is inapplicable as the attending circumstances are immaterial because the charge
constituting the complex crime have the corresponding penalty of reclusion perpetua.
Since the offense is punishable by reclusion perpetua, bail is discretionary. Instead of a
motion to fix bail, a summary hearing to determine if the evidence of guilt is strong is, therefore,
necessary conformably with Section 13, Article III of the 1987Constitution and Section 4, Rule
114 of the Rules.
Without filing a motion for reconsideration, petitioner elevated the matter before SC to resolve
the lone issue of whether an accused indicted for the complex crime of Malversation of Public
Funds thru Falsification ofOfficial/Public Documents involving an amount that exceeds
P22,000.00 is entitled to bail as a matter of right.
Issues:
Whether an accused indicted for the complex crime of Malversation of Public Funds thru
Falsification of

Official/Public Documents involving an amount that exceeds P22,000.00 is entitled to bail as


a matter of right.

Ruling:
No.
The controversy is, in fact, not one of first impression. Mañalac, Jr. v. People already
resolved that an accused charged with Malversation of Public Funds thru Falsification of
Official/Public Documents where the amount involved exceeds P22,000.00 is not entitled to bail
as a matter of right because it has an actual imposable penalty of reclusion perpetua.
In Mañalac, Jr., the defendants argued that they should be allowed to post bail since
reclusion perpetua is not the prescribed penalty for the offense but merely describes the penalty
actually imposed on account of the fraud involved. It was also posited that Article 48... of the
RPC applies "only after the accused has been convicted in a full-blown trial such that the court
is mandated to impose the penalty of the most serious crime," and that the reason for the
imposition of the penalty of the most serious offense is "only for the purpose of... determining
the correct penalty upon the application of the Indeterminate Sentence Law."
This Court, through the Third Division, however, denied the petition and resolved in the
affirmative the issue of whether the constitutional right to bail of an accused is restricted in...
cases whose imposable penalty ranges from reclusion temporal maximum to reclusion
perpetua.
To recall, the amounts involved in Pantaleon, Jr. were manifestly in excess of
P22,000.00. We opined that the Sandiganbayan correctly imposed the penalty of reclusion
perpetua and that the ISL is inapplicable since it is an indivisible penalty.
The rulings in Pantaleon, Jr. and analogous cases are in keeping with the provisions of
the RPC. Specifically, Article 48 of which states that in complex crimes, "the penalty for the most
serious crime shall be imposed, the same to be applied in its maximum period."
Thus, in Malversation of Public Funds thru Falsification of Official/Public Documents, the
prescribed penalties for malversation and falsification should be taken into account. Under the
RPC, the penalty for malversation of public funds or property if the amount involved exceeds
P22,000.00 shall be reclusion temporal in its maximum period to reclusion perpetua, aside from
perpetual special disqualification and a fine equal to the amount of the funds malversed or equal
to the total value of the property embezzled.
On the other hand, the penalty of prision mayor and a fine not to exceed P5,000.00 shall
be imposed for falsification committed by a public officer. Considering that malversation is the
more serious offense, the imposable penalty for
Malversation of Public Funds thru Falsification of Official/Public Documents if the amount
involved exceeds P22,000.00 is reclusion perpetua, it being the maximum period of the
prescribed penalty of reclusion temporal in its maximum period to... reclusion perpetua.
For purposes of bail application, however, the ruling in Mañalac, Jr. should be revisited
on the ground that Pantaleon, Jr. (as well as Conwi, Jr., Enfermo, Pajaro, et al., and Zafra) was
disposed in the context of a judgment of conviction rendered by the... lower court and affirmed
on appeal by this Court.
The Court held that the term "punishable" should refer to prescribed, not imposable,
penalty. Nonetheless, reading through the text of the assailed Resolution reveals that the anti-
graft court actually meant... prescribed penalty whenever it referred to imposable penalty.
Therefore, in essence, the ruling is correct. Respondent court held: If the complex crime
of Malversation thru Falsification be imposed in its maximum period, there is no doubt that, in
case of conviction, the penalty to be imposed is reclusion perpetua. The cases, however, are
still at their inception. Criminal... proceedings are yet to ensue. This is not the proper time,
therefore, to call for the application of the penalty contemplated under Article 48 by imposing the
same in its maximum period.
For purposes of determining whether a person can be admitted to bail as a matter of
right, it is the imposable penalty prescribed by law for the crime charged which should be
considered and, not the penalty to be actually imposed.

RELIANCE SURETY & INSURANCE CO., VS AMANTE


GR No. 150994, June 30, 2005

Doctrine: Any domestic or foreign corporation, licensed as a surety in accordance with law and
currently authorized to act as such, may provide bail by a bond subscribed jointly by the
accused and an officer of the corporation duly authorized by its board of directors. Once the
obligation of bail is assumed, the bondsman or surety becomes in law the jailer of the accused
and is subrogated to all the rights and means which the government possesses to make his
control of him effective.
Section 21, Rule 114 of the 1985 Rules of Criminal Procedure, in force at the time of the subject
incidents, provides for the procedure to be followed before a bail bond may be forfeited, and
judgment on the bond rendered against the surety:

SEC. 21. Forfeiture of bailbond. — When the presence of the accused is required by the court,
or these Rules, his bondsman shall be notified to produce him before the court on a given date.
If the accused fails to appear in person as required, the bond shall be declared forfeited and the
bondsman are given thirty (30) days within which to produce their principal and to show cause
why judgment should not be rendered against them for the amount of their bond. Within the said
period, the bondsmen:

(a) must produce the body of their principal or give the reason for his non-production; and

(b) must explain satisfactorily why the accused did not appear before the court when first
required to do so.

Failing in these two requisites, a judgment shall be rendered against the bondsmen, jointly and
severally, for the amount of the bond, and the court shall not reduce or otherwise mitigate the
liability of the bondsmen, except when the accused has been surrendered or is acquitted.

Facts:

Petitioner Reliance Surety & Insurance Co., Inc. (Reliance) is a duly organized insurance firm.
On 5 October 1998, it filed a Special Appearance And Motion to Set Aside Orders/Writs of
Execution with the Regional Trial Court (RTC) of Cabanatuan City, Branch 23, presided over by
Hon. Andres R. Amante, Jr. Reliance pertinently alleged therein, thus Reliance entered its
special appearance in each of the five (5) criminal cases, at the same time seeking to set aside
the cited writs of execution. Reliance alleged that the bonds in question were issued by one
Evelyn Tinio, against whom it had since lodged a criminal case.

Each of the criminal cases were prosecuted in behalf of the People of the Philippines by the City
Prosecutor, who did not interpose any objection to Reliance’s motion. Respondent judge
conducted a hearing on the matter, and Reliance submitted documentary evidence in support of
its motion.

On 21 April 1999, respondent judge issued an Order denying Reliance’s motion. On the premise
that the controversy revolved on the "tri-sided (sic) relationship of movant Reliance Surety;
Alfredo Wy and Evelyn Tinio and the Insurance Commission," the Order stressed that the
controversy "could only be resolved with authority and finality by the Insurance Commission
under its Administrative and Adjudicatory Powers."

As Reliance failed in its motion to reconsider the said Order, on 15 June 1999, it seasonably
filed a Notice of Appeal. However, on 15 July 1999, respondent judge issued an Order
disallowing the Notice of Appeal on the ground that Reliance failed "to pay the corresponding
appeal fee, pursuant to the provisions of Sec. 1 (c), Rule 50, in relation to Sec. 4, Rule 41."

Reliance sought the reconsideration of the disallowance of the appeal, stressing among others,
that the rules cited by the RTC were inapplicable, as they pertained to civil actions and not to
criminal cases, and that there was nothing in the Rules of Criminal Procedure that requires the
payment of appeal fees in criminal cases. However, Reliance’s Motion for Reconsideration was
denied in an Order dated 24 August 1999. Therein, the RTC characterized the pending incident
as having a "civil nature," which has not been subsumed by the criminal nature of the cases
under which Reliance’s motion was captioned.

Reliance then filed a Petition for Mandamus with the Court of Appeals, praying that the orders
disallowing the Notice of Appeal be declared null and void, and that respondent Judge be
ordered to immediately transmit the complete records, together with the Notice of Appeal in
accordance with Section 8, Rule 12 of the Rules of Court.

Issue:

1. Whether docket fees should be paid in appealing the order dismissing petitioner’s motion
to set aside order/writ of execution, the appellate court cited Section 7, Rule 5 of the
Revised Internal Rules of the Court of Appeals (RIRCA), which provides that "appeals
from orders of confiscation or forfeiture of bail bonds shall be treated as appeals in civil
cases.

2. Whether Section 3, Rule 5 of the Revised Internal Rules of the Court of Appeals
(RIRCA) which ordains that "no payment of docketing and other legal fees shall be
required in criminal cases except in petitions for review of criminal cases and appeals
from confiscation or forfeiture of bail bond.

Ruling:

The Court of Appeals is empowered to promulgate its own rules or orders pertaining to its
operations.40 The RIRCA is the by-product of this vested authority. The provisions which
Reliance assails, Sections 3 and 7 of Rule 5, were in place as early as 1994, when the RIRCA
was amended, and retained in the 1999 revision of the RIRCA.

Reliance cannot disavow knowledge of the provisions of the RIRCA. That these rules are called
"internal" does not mean that they are secret. In fact, both the 1994 and 1999 editions of the
RIRCA have been widely disseminated, available upon demand from the Court of Appeals, and
even replicated in private collations or annotations of our laws.41 Nor can Reliance validly
assert that the RIRCA provisions supplanted the Rules of Court. The RIRCA necessitated the
approval, which was obtained, of the Supreme Court prior to its effectivity, the very Supreme
Court which promulgated or amended our Rules of Court. Certainly, the Court cannot be
precluded from amending its own issuances, or issuing supplementary or clarificatory
procedural rules such as the RIRCA. Indeed, the RIRCA, vested with the requisite imprimatur of
the Supreme Court, is effectively an issuance of this Court. Challenge of the RIRCA is akin to
challenging one of the Court’s very own issuances.

Moreover, the assailed provisions of the RIRCA are ultimately correct in characterizing the
appeal from a judgment on the bail bond as inherently civil in nature. The RTC correctly picked
up on this point,42 and it is a characterization that we can affirm. The liability of the bondsmen
on the bail bond arises not from the violation of, or an obligation to comply with, a penal
provision of law. It emerges instead from a contract, the bond subscribed jointly by the accused
and the surety or bondsmen. The obligation of the accused on the bond is different from that of
the surety in that the former can be made to suffer a criminal penalty for failure to comply with
the obligations on the bail bond. However, the surety is not under a similar pain of punishment,
as its liability on the bail bond would merely be civil in character. Nothing in the Rules of Court
authorizes the imprisonment of the surety for the failure to produce the accused when called for
in court, his obligation being contractual in source and character. In keeping with the civil nature
of the appeal from the judgment on the bail bonds, the Court of Appeals acted properly in
prescribing the payment of docket fee for such appeal as in appeal in civil cases.

Reliance apparently failed to show cause to the RTC why it should not be held liable on the
subject bail bonds, to timely appeal the judgment rendered on the bail bonds, or to move within
a reasonable time to set aside the writs of execution. When Reliance finally undertook steps to
acquit itself of liability on these bail bonds but faced denial of its relief by the RTC, it failed to
lodge the correct mode of judicial review when it filed a notice of appeal instead of a special civil
action for certiorari. From the disallowance of the Notice of Appeal, it responded with a woefully
insufficient petition for mandamus. Even its arguments against the validity of the questioned
RIRCA provisions are erroneous.

In a long-distance race, the crowd would be charitably disposed the first time a runner trips and
falls. Neither would the second fall exhaust the commiseration of the spectators. However, if the
runner stumbles every fifty meters, observers have earned the right to heckle and jeer, or more
kindly, to question whether the racer is qualified to run in the first place. Indeed, Reliance’s
consecutive procedural missteps, all of which could have been avoided by easy reference to the
established rules and jurisprudence, have deprived it of the right to seek relief before this Court.

Our final disposition is not the product of sheer pique, and we have duly considered the fact that
denial of the petition would condemn Reliance to an obligation it might not have contracted. Yet
ultimately, it should accept the consequences of its negligence in failing to timely present its
position, or in utilizing the proper modes of judicial review. Equitable relief is not the supremacy
of pity but the entitlement of due process previously denied the litigant. One who fails to avail of
the prescribed legal steps despite repeated opportunities has no reason to expect anything
other than due condemnation.

WHEREFORE, the Petition is DENIED. Costs against petitioner.

EFRAIM GENUINO VS LEILA DE LIMA


G.R. No. 197930, April 17, 2018, Reyes Jr.

DOCTRINE: “The point is that in the conduct of a preliminary investigation, the presence of the
accused is not necessary for the prosecutor to discharge his investigatory duties. If the accused
chooses to waive his presence or fails to submit countervailing evidence, that is his own
lookout. Ultimately, he shall be bound by the determination of the prosecutor on the presence of
probable cause and he cannot claim denial of due process.The DOJ therefore cannot justify the
restraint in the liberty of movement imposed by DOJ Circular No. 41 on the ground that it is
necessary to ensure presence and attendance in the preliminary investigation of the complaints.
There is also no authority of law granting it the power to compel the attendance of the subjects
of a preliminary investigation, pursuant to its investigatory powers under E.O. No. 292. Its
investigatory power is simply inquisitorial and, unfortunately, not broad enough to embrace the
imposition of restraint on the liberty of movement.”

FACTS:

These consolidated Petitions for Certiorari and Prohibition with Prayer for the Issuance of
Temporary Restraining Orders (TRO) and/or Writs of Preliminary Injunction under Rule 65 of the
Rules of Court assail the constitutionality of Department of Justice (DOJ) Circular No. 41, series
of 2010, otherwise known as the Consolidated Rules and Regulations Governing Issuance and
Implementation of Hold Departure Orders, Watchlist Orders and Allow Departure Orders, on the
ground that it infringes on the constitutional right to travel. On May 25, 2010, then Acting DOJ
Secretary Alberto C. Agra issued the assailed DOJ Circular No. 41, consolidating DOJ Circular
Nos. 17 and 18, which govern the issuance and implementation of HDOs, WLOs, and ADOs.
After the expiration of GMA’s term as President of the Republic of the Philippines and her
subsequent election as Pampanga representative, criminal complaints were filed against her
before the DOJ particularly plunder, malversation and/or illegal use of OWWA funds, illegal use
of public funds, graft and corruption, violation of the OEC, violation of the Code of Conduct on
Ethical Standards for Public Officials and qualified theft. In view of the foregoing criminal
complaints, De Lima issued DOJWLO No. 2011-422 against GMA pursuant to her authority
under DOJ Circular No.41. She also ordered for the inclusion of GMA’s name in the Bureau of
Immigration(BI) watchlist.On October 20, 2011, two criminal complaints for Electoral Sabotage
and Violation of the OEC were filed against GMA and her husband, Jose Miguel Arroyo.
Following the filing of criminal complaints, De Lima issued DOJ WLO No. 2011-573 against
GMA and Miguel Arroyo with a validity period of 60 days, unless sooner terminated or otherwise
extended. Meanwhile, in G.R. No. 197930, HDO No. 2011-64 was issued against Genuinos,
among others, after criminal complaints for Malversation and Violation of Sections3(e), (g), (h)
an (i) of R.A. No. 3019. The petitioners therein seek to annul and set asidethe following orders
issued by the former Secretary Leila De Lima, pursuant to the said circular.

ISSUE:
Whether or not the issuance of DOJ Circular No. 41 has a legal basis?

RULING:

Guided by the foregoing disquisition, the Court is in quandary of identifying the authority from
which the DOJ believed its power to restrain the right to travel emanates. To begin with, there is
no law particularly providing for the authority of the secretary of justice to curtail the exercise of
the right to travel, in the interest of national security, public safety or public health. As it is, the
only ground of the former DOJ Secretary in restraining the petitioners, at that time, was the
pendency of the preliminary investigation of the Joint DOJ-COMELEC Preliminary Investigation
Committee on the complaint for electoral sabotage against them.

To be clear, DOJ Circular No. 41 is not a law. It is not a legislative enactment which underwent
the scrutiny and concurrence of lawmakers, and submitted to the President for approval. It is a
mere administrative issuance apparently designed to carry out the provisions of an enabling law
which the former DOJ Secretary believed to be Executive Order (E.O.) No. 292, otherwise
known as the "Administrative Code of 1987.
It is, however, important to stress that before there can even be a valid administrative issuance,
there must first be a showing that the delegation of legislative power is itself valid. It is valid only
if there is a law that (a) is complete in itself, setting forth therein the policy to be executed,
carried out, or implemented by the delegate; and (b) fixes a standard the limits of which are
sufficiently determinate and determinable to which the delegate must conform in the
performance of his functions.
The DOJ stresses the necessity of the restraint imposed in DOJ Circular No. 41 in that to allow
the petitioners, who are under preliminary investigation, to exercise an untrammelled right to
travel, especially when the risk of flight is distinctly high will surely impede the efficient and
effective operation of the justice system. The absence of the petitioners, it asseverates, would
mean that the farthest criminal proceeding they could go would be the filing of the criminal
information since they cannot be arraigned in absentia.
The predicament of the DOJ is understandable yet untenable for relying on grounds other what
is permitted within the confines of its own power and the nature of preliminary investigation
itself. The Court, in Paderanga vs. Drilon,106 made a clarification on the nature of a preliminary
investigation, thus:
A preliminary investigation is x x x an inquiry or proceeding for the purpose of determining
whether there is sufficient ground to engender a well founded belief that a crime cognizable by
the Regional Trial Court has been committed and that the respondent is probably guilty thereof,
and should be held for trial. X X X A preliminary investigation is not the occasion for the full and
exhaustive display of the parties' evidence; it is for the presentation of such evidence only as
may engender a well grounded belief that an offense has been committed and that the accused
is probably guilty thereof.
It bears emphasizing that the conduct of a preliminary investigation is an implement of due
process which essentially benefits the accused as it accords an opportunity for the presentation
of his side with regard to the accusation. The accused may, however, opt to waive his presence
in the preliminary investigation. In any case, whether the accused responds to a subpoena, the
investigating prosecutor shall resolve the complaint within 10 days after the filing of the same.
The point is that in the conduct of a preliminary investigation, the presence of the accused is not
necessary for the prosecutor to discharge his investigatory duties. If the accused chooses to
waive his presence or fails to submit countervailing evidence, that is his own lookout. Ultimately,
he shall be bound by the determination of the prosecutor on the presence of probable cause
and he cannot claim denial of due process.
The DOJ therefore cannot justify the restraint in the liberty of movement imposed by DOJ
Circular No. 41 on the ground that it is necessary to ensure presence and attendance in the
preliminary investigation of the complaints. There is also no authority of law granting it the
power to compel the attendance of the subjects of a preliminary investigation, pursuant to its
investigatory powers under E.O. No. 292. Its investigatory power is simply inquisitorial and,
unfortunately, not broad enough to embrace the imposition of restraint on the liberty of
movement.
Liberty under the foregoing clause includes the right to choose one's residence, to leave it
whenever he pleases and to travel wherever he wills.
It is apparent, however, that the right to travel is not absolute. There are constitutional, statutory
and inherent limitations regulating the right to travel. Section 6 itself provides that the right to
travel may be impaired only in the interest of national security, public safety or public health, as
may be provided by law.
Clearly, under the provision, there are only three considerations that may permit a restriction on
the right to travel: national security, public safety or public health. As a further requirement,
there must be an explicit provision of statutory law or the Rules of Court[80] providing for the
impairment.
It is clear from the foregoing that the liberty of abode may only be impaired by a lawful order of
the court and, on the one hand, the right to travel may only be impaired by a law that concerns
national security, public safety or public health. Therefore, when the exigencies of times call for
a limitation on the right to travel, the Congress must respond to the need by explicitly providing
for the restriction in a law. This is in deference to the primacy of the right to travel, being a
constitutionally-protected right and not simply a statutory right, that it can only be curtailed by a
legislative enactment.
Gwendolyn Garcia v. Sandiganbayan
GR No. 205904-06, October 17, 2018

DOCRTRINE:
Upon posting bail, the accused subjects himself to the jurisdiction of the court and may validly
be restricted in his movement and prohibited from leaving this jurisdiction

FACTS:
Petitioner was charged with violation of Sec 3e and 3g of R.A. No. 3019 and Art. 220 of the
RPC filed before the Sandiganbayan. Sandigan also issued three hold departure orders against
the petitioner and her co-accused. Petitioner voluntarily surrendered and paid her three
separate bail on the amount of P30,000.00 for the three cases filed against her.

Petitioner received the hold departure order. She filed a motion for reconsideration questioning
the HDO as such cannot be issued without final determination of probable cause. She argued
that the HDO is violative of her constitutional right to travel, which may be impaired only in the
interest of national security, public safety, or public health, as maybe provided by law. She
argues that there should be an actual case filed and pending with the court before and HDO can
be issued. Since there was no final resolution yet on her motion for reconsideration, it cannot be
said that there is already a pending criminal case against her. The issuance of the HDOs,
therefore, was premature.

Sandiganbayan denied the petition hence the case before the court.

ISSUE:
Whether the HDO was premature.

RULING:
Upon posting bail, the accused subjects himself to the jurisdiction of the court and may validly
be restricted in his movement and prohibited from leaving this jurisdiction. He cannot leave the
country without the permission of the court where his case is pending. Remember that the grant
of bail merely secures provisional or temporary liberty under conditions set by the court. The
court may recall said grant and return the accused to detention should he violate the conditions
for his temporary liberty or when reasons for the lifting of his bail arise. Thus, it is not entirely
correct for the petitioner to argue that the issuance of HDOs amounted to an unreasonable
restriction on her liberty of movement or right to travel. The truth of the matter is that she was
already under restricted right to travel when she submitted to the jurisdiction of the
Sandiganbayan by posting bail. The rule is that "a person facing a criminal indictment and
provisionally released on bail does not have an unrestricted right to travel, the reason being that
a person's right to travel is subject to the usual constraints imposed by the very necessity of
safeguarding the system of justice." The issuance of the HDO is a process complementary to
the granting of bail since it puts the Bureau of Immigration on notice that a certain person is
charged before the courts of law and must not be allowed to leave our jurisdiction without the
permission of the court. After all, the granting of bail does not guaranty compliance by the
accused of the conditions for his temporary liberty, particularly, his presence at every stage of
the proceedings. Some, if not all, maybe tempted to jump bail and leave the country. This is
what the HDO seeks to avoid by keeping the accused within the territory where court processes
and dispositions may be enforced and implemented.

CYRIL CALPITO QUI, Petitioner, v. PEOPLE OF THE PHILIPPINES, Respondent.


G.R. No. 196161 : September 26, 2012

DOCTRINE: Bail pending appeal is governed by Sec. 5 of Rule 114, Revised Rules of Criminal
Procedure, which provides: Sec. 5. Bail, when discretionary. Upon conviction by the Regional
Trial Court of an offense not punishable by death, reclusion perpetua, or life imprisonment,
admission to bail is discretionary.

The grant of bail is a matter of discretion upon conviction by the RTC of an offense not
punishable by death, reclusion perpetua or life imprisonment Certainly, after one is convicted by
the trial court, the presumption of innocence, and with it, the constitutional right to bail, ends.

FACTS: Petitioner was charged with two counts of violation of Section 10(a), Article VI of
Republic Act No. (RA) 7610 or the Special Protection of Children Against Child Abuse,
Exploitation and Discrimination Act for committing acts of cruelty and child abuse upon Christian
John Ignacio, a minor. RTC convicted petitioner as charged, and sentenced her to two equal
periods of imprisonment for an indeterminate penalty of five (5) years, four (4) months and
twenty one (21) days of prision correccional in its maximum period, as minimum, to seven (7)
years, four (4) months and one (1) day of prision mayor in its minimum period, as maximum.

With the perfection of her appeal and the consequent elevation of the case records to the CA,
petitioner posthaste filed before the appellate court an Urgent Petition/Application for Bail
Pending Appeal which respondent People of the Philippines, through the Office of the Solicitor
General (OSG), opposed. The OSG urged for the denial of the bail application on the ground of
petitioners propensity to evade the law and that she is a flight-risk, as she in fact failed to attend
several hearings before the RTC resulting in the issuance of three warrants for her arrest. CA
denied petitioners application for bail pending appeal on the basis of Sec. 5(d) of Rule 114,
Revised Rules of Criminal Procedure.

ISSUE: Is the accused entitled to the right to bail?

HELD: No. Bail pending appeal is governed by Sec. 5 of Rule 114, Revised Rules of Criminal
Procedure, which provides: Sec. 5. Bail, when discretionary. Upon conviction by the Regional
Trial Court of an offense not punishable by death, reclusion perpetua, or life imprisonment,
admission to bail is discretionary.
If the penalty imposed by the trial court is imprisonment exceeding six (6) years, the accused
shall be denied bail, or his bail shall be cancelled upon a showing by the prosecution, with
notice to the accused, of the following or other similar circumstances:

(a) That he is a recidivist, quasi-recidivist, or habitual delinquent, or has committed the crime
aggravated by the circumstance of reiteration;

(b) That he has previously escaped from legal confinement, evaded sentence, or violated the
conditions of his bail without a valid justification;

(c) That he committed the offense while under probation, parole, or conditional pardon;

(d) That the circumstances of his case indicate the probability of flight if released on bail;
or

(e) That there is undue risk that he may commit another crime during the pendency of the
appeal.

The appellate court may, motu proprio or on motion of any party, review the resolution of the
Regional Trial Court after notice to the adverse party in either case. (Emphasis supplied.)

Under the present rule, the grant of bail is a matter of discretion upon conviction by the RTC of
an offense not punishable by death, reclusion perpetua or life imprisonment, as here. Indeed,
pursuant to the "tough on bail pending appeal" policy, the presence of bail-negating conditions
mandates the denial or revocation of bail pending appeal such that those circumstances are
deemed to be as grave as conviction by the trial court for an offense punishable by death,
reclusion perpetua or life imprisonment where bail is prohibited. ‚rνll

The CA properly exercised its discretion in denying petitioners application for bail pending
appeal. The CAs determination as to petitioner being a high risk for flight is not without factual
mooring. Indeed, the undisputed fact that petitioner did not attend the hearings before the RTC,
which compelled the trial court to issue warrants for her arrest, is undeniably indicative of
petitioners propensity to trifle with court processes. This fact alone should weigh heavily against
a grant of bail pending appeal. Petitioners penchant to disobey court processes may also be
deduced from the fact that she lied in order to wiggle out of, and justify her non-appearance on
the March 8, 2010 hearing before the RTC. Petitioner gave the convenient but false excuse that
her father, Cirilo Calpito, was hospitalized on said hearing day (i.e., March 8, 2010) and that
Cirilo died on March 24, 2010. The lies foisted on the court were exposed by: (1) the Death
Certificate of Cirilo Calpito clearly showing that he died on March 24, 2009 or a year before the
aforesaid March 2010 RTC hearing; and (2) the Certification issued by Dr. Aniana Javier stating
that Cirilo went to her clinic on March 9, 2009. The fact of transferring residences without
informing her bondsman and the trial court can only be viewed as petitioners inclination to
evade court appearance, as indicative of flight, and an attempt to place herself beyond the pale
of the law.

Petitioners argument that she has the constitutional right to bail and that the evidence of guilt
against her is not strong is spurious. Certainly, after one is convicted by the trial court, the
presumption of innocence, and with it, the constitutional right to bail, ends. In all, the Court finds
the CA to have exercised its discretion soundly when it denied petitioner's application for bail
pending appeal.ςηαÎ
GOVERNMENT OF HONG KONG SPECIAL ADMINISTRATIVE REGION, represented by the
Philippine Department of Justice, petitioner, vs. HON. FELIXBERTO T. OLALIA, JR. and
JUAN ANTONIO MUÑOZ, respondents.

G.R. No. 153675. April 19, 2007.

DOCTRINE: An extradition proceeding being sui generis, the standard of proof required in granting or
denying bail can neither be the proof beyond reasonable doubt in criminal cases nor the standard of
proof of preponderance of evidence in civil cases—the potential extraditee must prove by “ clear and
convincing proof” that he is not a flight risk and will abide with all the orders and processes of the
extradition court

FACTS: On January 30, 1995, the Republic of the Philippines and British Clown Colony of
Hongkong, which later became Hongkong Special Administrative Region, entered into an
“Agreement for the Surrender of Accused and Convicted Persons.” It took effect in 1997.

In 1999, Juan Antonio Muñoz was charged before the Hong Kong Court with 3 counts of
accepting an advantage as agent and 7 counts of conspiracy to defraud in which, if convicted,
he will face 7 to 14 years for each charge. Subsequently, the DOJ received from the Hong Kong
Department of Justice a request for the provisional arrest of private respondent. The request
was then forwarded to the RTC in Manila which issued the order of arrest.

Petitioner, Hong Kong Special Administrative Region, then filed with the RTC of Manila a
petition for the extradition of Muñoz. In that same case, a petition for bail was filed by Munoz.
After two (2) years, Hon. Olalia, the respondent judge, granted the petition for bail filed by
Muñoz. Hence, the instant Petition for Certiorari under Rule 65 filed by the petitioner seeking to
nullify the said order granting the application for bail.

ISSUE: Whether or not bail may be granted in extradition proceedings?

RULING: The Court ruled in the affirmative.

In the case of Government of United States of America v. Hon. Guillermo G. Purganan, it ruled
that bail is not available in extradition proceedings and is only available in criminal proceedings.
However, such ruling must be re-examined because of the modern trend in public international
law wherein primacy is placed on the worth of every individual and the sanctity of human rights.

In the cases of US v. Go-Sioco and Mejoff v. Director of Prisons, basing both rulings on
Universal declaration of Human Rights, held that the right to bail may be granted in deportation
proceedings.

If bail can be granted in deportation cases, we see no justification why it should not also be
allowed in extradition cases. Likewise, considering that the Universal Declaration of Human
Rights applies to deportation cases, there is no reason why it cannot be invoked in extradition
cases. After all, both are administrative proceedings where the innocence or guilt of the person
detained is not in issue.

Extradition has been characterized as the right of a foreign power, created by treaty, to demand
the surrender of one accused or convicted of a crime within its territorial jurisdiction, and the
correlative duty of the other state to surrender him to the demanding state. It is not a criminal
proceeding. It is not a trial to determine the guilt or innocence of the potential extraditee. Nor is
it a full-blown civil action, but one that is merely administrative in character.

Muñoz had been detained for over two (2) years without having been convicted of any crime. By
any standard, such an extended period of detention is a serious deprivation of his fundamental
right to liberty. In fact, it was this prolonged deprivation of liberty which prompted the extradition
court to grant him bail. While our extradition law does not provide for the grant of bail to an
extraditee, however, there is no provision prohibiting him or her from filing a motion for bail, a
right to due process under the Constitution.

The applicable standard of due process, however, should not be the same as that in criminal
proceedings. In the latter, the standard of due process is premised on the presumption of
innocence of the accused. Bearing in mind the purpose of extradition proceedings, the premise
behind the issuance of the arrest warrant and the “temporary detention” is the possibility of flight
of the potential extraditee. This is based on the assumption that such extraditee is a fugitive
from justice. Given the foregoing, the prospective extraditee thus bears the onus probandi
of showing that he or she is not a flight risk and should be granted bail.

An extradition proceeding being sui generis, the standard of proof required in granting or
denying bail can neither be the proof beyond reasonable doubt in criminal cases nor the
standard of proof of preponderance of evidence in civil cases. While administrative in character,
the standard of substantial evidence used in administrative cases cannot likewise apply given
the object of extradition law which is to prevent the prospective extraditee from fleeing our
jurisdiction. In his Separate Opinion in Purganan, then Associate Justice, now Chief Justice
Reynato S. Puno, proposed that a new standard which he termed “clear and convincing
evidence” should be used in granting bail in extradition cases. According to him, this standard
should be lower than proof beyond reasonable doubt but higher than preponderance of
evidence. The potential extraditee must prove by “clear and convincing evidence” that he is
not a flight risk and will abide with all the orders and processes of the extradition court.

In this case, Muñoz was not able to present evidence to show that he is not a flight risk. Hence,
this case was remanded to the trial court to determine whether he should be granted bail on the
basis of “clear and convincing evidence”.

JUAN PONCE ENRILE v. SANDIGANBAYAN (THIRD DIVISION), AND PEOPLE OF THE


PHILIPPINES,
G.R. No. 213847, August 18, 2015

DOCTRINES: Primary objective of bail – The strength of the Prosecution's case, albeit a good
measure of the accused's propensity for flight or for causing harm to the public, is subsidiary to
the primary objective of bail, which is to ensure that the accused appears at trial.

Bail is a right and a matter of discretion – Right to bail is afforded in Sec. 13, Art III of the 1987
Constitution and repeated in Sec. 7, Rule 114 of the Rules of Criminal Procedure to wit: “No
person charged with a capital offense, or an offense punishable by reclusion perpetua or life
imprisonment, shall be admitted to bail when evidence of guilt is strong, regardless of the stage
of the criminal prosecution.”

FACTS: The Office of the Ombudsman charged Enrile, 90 years of age, and several others with
plunder in the Sandiganbayan on the basis of their purported involvement in the diversion and
misuse of appropriations under the Priority Development Assistance Fund (PDAF). Upon
voluntary surrender, Enrile filed his Motion for Detention at the PNP General Hospital, and his
Motion to Fix Bail. Enrile claims that before judgment of conviction, an accused is entitled to bail
as matter of right; that it is the duty and burden of the Prosecution to show clearly and
conclusively that Enrile comes under the exception and cannot be excluded from enjoying the
right to bail; that the Prosecution has failed to establish that Enrile, if convicted of plunder, is
punishable by reclusion perpetua considering the presence of two mitigating circumstances –
his age and his voluntary surrender; that the Prosecution has not come forward with proof
showing that his guilt for the crime of plunder is strong; and that he should not be considered a
flight risk taking into account that he is already over the age of 90, his medical condition, and his
social standing. In its Comment, the Ombudsman contends that Enrile’s right to bail is
discretionary as he is charged with a capital offense; that to be granted bail, it is mandatory that
a bail hearing be conducted to determine whether there is strong evidence of his guilt, or the
lack of it; and that entitlement to bail considers the imposable penalty, regardless of the
attendant circumstances.

ISSUE: Whether or not Enrile is entitle to bail. If so, on what ground(s)?

HELD: Yes, Enrile is entitled to bail as a matter of right based on humanitarian grounds.

The decision whether to detain or release an accused before and during trial is ultimately an
incident of the judicial power to hear and determine his criminal case. The strength of the
Prosecution’s case, albeit a good measure of the accused’s propensity for flight or for causing
harm to the public, is subsidiary to the primary objective of bail, which is to ensure that the
accused appears at trial.

The Court is guided by the earlier mentioned principal purpose of bail, which is to guarantee the
appearance of the accused at the trial, or whenever so required by the court. The Court is
further mindful of the Philippines’ responsibility in the international community arising from the
national commitment under the Universal Declaration of Human Rights to:

x x x uphold the fundamental human rights as well as value the worth and dignity of every
person. This commitment is enshrined in Section II, Article II of our Constitution which provides:
“The State values the dignity of every human person and guarantees full respect for human
rights.” The Philippines, therefore, has the responsibility of protecting and promoting the right of
every person to liberty and due process, ensuring that those detained or arrested can
participate in the proceedings before a court, to enable it to decide without delay on the legality
of the detention and order their release if justified. In other words, the Philippine authorities are
under obligation to make available to every person under detention such remedies which
safeguards their fundamental right to liberty. These remedies include the right to be admitted to
bail.

This national commitment to uphold the fundamental human rights as well as value the worth
and dignity of every person has authorized the grant of bail not only to those charged in criminal
proceedings but also to extraditees upon a clear and convincing showing: (1) that the detainee
will not be a flight risk or a danger to the community; and (2 ) that there exist special,
humanitarian and compelling circumstances.

In our view, his social and political standing and his having immediately surrendered to the
authorities upon his being charged in court indicate that the risk of his flight or escape from this
jurisdiction is highly unlikely. His personal disposition from the onset of his indictment for
plunder, formal or otherwise, has demonstrated his utter respect for the legal processes of this
country. We also do not ignore that at an earlier time many years ago when he had been
charged with rebellion with murder and multiple frustrated murder, he already evinced a similar
personal disposition of respect for the legal processes, and was granted bail during the
pendency of his trial because he was not seen as a flight risk. With his solid reputation in both
his public and his private lives, his long years of public service, and history’s judgment of him
being at stake, he should be granted bail.

SERAPIO VS. SANDIGANBAYAN,


January 28, 2003

DOCTRINE: Petition for habeas corpus is not the appropriate remedy for asserting ones right to
bail. It cannot be availed of where accused is entitled to bail not as a matter of right but on the
discretion of the court and the latter has not abused such discretion in refusing to grant bail, or
has not even exercised said discretion. The proper recourse is to file an application for bail with
the court where the criminal case is pending and to allow hearings thereon to proceed.

FACTS: 
Atty. Edward Serapio (petitioner) filed two petitions in the SC; these are: A petition for certiorari
assailing the resolutions of the Third division of the Sandiganbayan denying his petition for bail,
motion for reinvestigation and motion to quash; and Petition for Habeas Corpus. Petitioner was
charged with the crime of plunder together with Former President Joseph Estrada and son
Jinggoy Estrada among others. Petitioner was a member of the Board of Trustees and legal
counsel of Erap Muslim Youth Foundation. He allegedly received, on behalf of the said
foundation, millions of pesos coming from illegal activities. The Ombudsman recommended the
filing of a case against him before the Sandiganbayan. A warrant for his arrest was issued.
Upon learning of the said warrant he voluntarily surrendered to the PNP. Petitioner, thereafter,
file an Urgent Motion for Bail but such motion is opposed by the prosecution for the reason that
petitioner should be arraign first before he can avail of Bail. Later on Petitioner simultaneously
filed a motion to quash.
The bail hearing was reset several times due to various pleadings filed by petitioner and the
prosecution. Due to this, petitioner filed a petition for habeas corpus for the reason that the
prosecution have waived their right to present evidence in opposition to his petition for bail; the
prosecution launched an endless barrage of obstructive and dilatory moves to prevent the
conduct of the bail hearings; and, on the failure of the People to adduce strong evidence of his
guilt. For the said reasons, he is still being deprived of his liberty. Petitioner cited also Moncupa
vs. Enrile, which in such case the Court held that habeas corpus extends to instances where
detention, while valid from its inception, has later become arbitrary.
ISSUE: 

Whether the petition habeas corpus should be granted?

RULING:

  No. SC finds no basis for the issuance of the writ of habeas corpus. General rule
applies. “Petition for habeas corpus is not the appropriate remedy for asserting ones right to
bail. It cannot be availed of where accused is entitled to bail not as a matter of right but on the
discretion of the court and the latter has not abused such discretion in refusing to grant bail, or
has not even exercised said discretion. The proper recourse is to file an application for bail with
the court where the criminal case is pending and to allow hearings thereon to proceed.”
When bail is a matter of right, an accused may apply for and be granted bail even prior to
arraignment. Moncupa vs Enrile does not apply in this case because petitioner’s restraint of
liberty did not become arbitrary. His application for bail has yet to commence. The delay in the
hearing of his petition for bail cannot be pinned solely to the Sandiganbayan or on the
prosecution because he himself is partly to be blamed. As a general rule, the writ of habeas
corpus will not issue where the person alleged to be restrained of his liberty in custody of an
officer under a process issued by the court which jurisdiction to do so.
In exceptional circumstances, habeas corpus may be granted by the courts even when the
person concerned is detained pursuant to a valid arrest or his voluntary surrender, for this writ of
liberty is recognized as the fundamental instrument for safeguarding individual freedom against
arbitrary and lawless state action due to its ability to cut through barriers of form and procedural
mazes.

RENATO M. DAVID vs. EDITHA A. AGBAY and PEOPLE OF THE PHILIPPINES

G. R. No. 199113

Doctrine: Custody of the law is required before the court can act upon the application for bail, but
is not required for the adjudication of other reliefs sought by the defendant where the mere application
therefor constitutes a waiver of the defense of lack of jurisdiction over the person of the accused.

FACTS: Petitioner migrated to Canada where he became a Canadian citizen by naturalization. Upon
their retirement, petitioner and his wife returned to the Philippines. Sometime in 2000, they purchased
a 600-square meter lot along the beach in Tambong, Gloria, Oriental Mindoro where they constructed a
residential house. Petitioner filed a Miscellaneous Lease Application (MLA) over the subject land with
the Department of Environment and Natural Resources (DENR).  In the said application, petitioner
indicated that he is a Filipino citizen.
Private respondent Editha A. Agbay opposed the application on the ground that petitioner, a Canadian
citizen, is disqualified to own land. She also filed a criminal complaint for falsification of public
documents under Article 172 of the Revised Penal Code (RPC). In his defense, petitioner averred that at
the time he filed his application, he had intended to re-acquire Philippine citizenship and that he had
been assured by a CENRO officer that he could declare himself as a Filipino.

The Office of the Provincial Prosecutor issued its Resolution finding probable cause to indict petitioner
for violation of Article 172 of the RPC and recommending the filing of the corresponding information in
court. Petitioner challenged the said resolution in a petition for review he filed before the Department
of Justice (DOJ) which was denied. The petitioner filed an Urgent Motion for Re-Determination of
Probable Cause10 in the MTC which was also denied due to lack of jurisdiction over the person of the
accused, and for lack of merit. Petitioner elevated the case to the RTC via a petition15 for certiorari
under Rule 65, alleging grave abuse of discretion on the part of the MTC. He asserted that first,
jurisdiction over the person of an accused cannot be a pre-condition for the re-determination of
probable cause by the court that issues a warrant of arrest.

ISSUE: Whether Jurisdiction over the person is necessary for Motion for re-determination of probable
cause of arrest

Held: No.

Custody of the law is not required for the adjudication of reliefs other than an application for bail. Court
discussed the distinction between custody of the law and jurisdiction over the person, and held that
jurisdiction over the person of the accused is deemed waived when he files any pleading seeking an
affirmative relief, except in cases when he invokes the special jurisdiction of the court by impugning
such jurisdiction over his person.

The voluntary appearance of the accused, whereby the court acquires jurisdiction over his person, is
accomplished either by his pleading to the merits (such as by filing a motion to quash or other pleadings
requiring the exercise of the court’s jurisdiction thereover, appearing for arraignment, entering trial) or
by filing bail. On the matter of bail, since the same is intended to obtain the provisional liberty of the
accused, as a rule the same cannot be posted before custody of the accused has been acquired by the
judicial authorities either by his arrest or voluntary surrender.
Custody of the law is required before the court can act upon the application for bail, but is not required
for the adjudication of other reliefs sought by the defendant where the mere application therefor
constitutes a waiver of the defense of lack of jurisdiction over the person of the accused. Custody of the
law is accomplished either by arrest or voluntary surrender, while jurisdiction over the person of the
accused is acquired upon his arrest or voluntary appearance. One can be under the custody of the law
but not yet subject to the jurisdiction of the court over his person, such as when a person arrested by
virtue of a warrant files a motion before arraignment to quash the warrant. On the other hand, one can
be subject to the jurisdiction of the court over his person, and yet not be in the custody of the law, such
as when an accused escapes custody after his trial has commenced. Being in the custody of the law
signifies restraint on the person, who is thereby deprived of his own will and liberty, binding him to
become obedient to the will of the law. Custody of the law is literally custody over the body of the
accused. It includes, but is not limited to, detention.

Villanueva vs. Buaya, November 22, 2010

Doctrine:

The nature of a bail hearing in petitions for bail. Where bail is a matter of discretion, the grant or
the denial of bail hinges on the issue of whether or not the evidence on the guilt of the accused
is strong and the determination of whether or not the evidence is strong is a matter of judicial
discretion which remains with the judge. In order for the judge to properly exercise this
discretion, he must first conduct a hearing to determine whether the evidence of guilt is strong.
This discretion lies not in the determination of whether or not a hearing should be held, but in
the appreciation and evaluation of the weight of the prosecution’s evidence of guilt against the
accused.

Facts:

On September 27, 2004, Assistant Provincial Prosecutor Prudencio O. Borgueta, Jr. issued a
Joint Resolution on Review, of R.A. No. 7610 against the accused. He likewise recommended
the cancellation of the bail bond of ₱100,000.00 (per case) posted by Tupa as, under Section
31, Article XII of R.A. No. 7610, if the offender is a public officer or employee, the penalty
provided in Section 5, Article III of R.A. No. 7610 is imposed in the maximum period,
i.e., reclusion perpetua. Thus, bail is not a matter of right. He also added that the cancellation of
the bail bond was all the more appropriate since there was strong evidence of guilt against the
accused based on Villanueva’s affidavit-complaint and her material declarations during the
preliminary investigation. The accused did not refute these declarations and, in fact, even
admitted the alleged sexual acts in his counter-affidavit and through his statements during the
clarificatory hearing. 

On the very same day (December 8, 2004), Tupa allegedly surrendered voluntarily to SPO2
Charito Daau of the Ormoc City Police Station and filed with the RTC, Branch 17 an Urgent Ex-
Parte Motion to Grant Bail (ex-parte motion).

In an Order issued on the same day the ex-parte motion was filed, without hearing and without
notice to the prosecution, Judge Buaya granted the ex-parte motion and ordered the release of
Tupa on bail.

On December 16, 2004, Villanueva moved to reconsider the order granting the ex-parte motion.
She argued that an application for bail should be heard and cannot be contained in a mere ex-
parte motion. Judge Buaya noted that Villanueva’s motion for reconsideration was submitted by
the private prosecutor without the conformity of the public prosecutor, as required under the
Rules on Criminal Procedure. Without acting on the merits of the said motion, Judge Buaya
issued an order allowing the accused to submit his comment or opposition within ten days;
thereafter, the matter would be submitted for resolution.

Judge Buaya’s differing treatment of the ex-parte motion and her motion for reconsideration
apparently irked Villanueva, prompting her to file the present administrative complaint against
the RTC judge. She observed the seeming bias and unfairness of Judge Buaya’s orders when
he granted the ex-parte motion without the required notice and hearing; on the other hand, he
did not act on her motion for reconsideration because it was not in the proper form, but allowed
the accused to comment on her motion.

On December 16, 2004, Villanueva moved to reconsider the order granting the ex-parte motion.
She argued that an application for bail should be heard and cannot be contained in a mere ex-
parte motion. Judge Buaya noted that Villanueva’s motion for reconsideration was submitted by
the private prosecutor without the conformity of the public prosecutor, as required under the
Rules on Criminal Procedure. 

Issue:

Whether or not bail was a matter of right in the present case is judicial in nature. 

Held:
Cases already decided on the matter of bail, we feel justified to expect judges to diligently
discharge their duties on the grant or denial of applications for bail. Basco v. Rapatalo aid down
the rules outlining the duties of a judge in case an application for bail is filed:

(1) Notify the prosecutor of the hearing of the application for bail or require him to submit his
recommendation x x x;

(2) Conduct a hearing of the application for bail regardless of whether or not the prosecution
refuses to present evidence to show that the guilt of the accused is strong for the purpose of
enabling the court to exercise its discretion x x x;

(3) Decide whether the evidence of guilt of the accused is strong based on the summary of
evidence of the prosecution x x x; [and]

(4) If the guilt of the accused is not strong, discharge the accused upon the approval of the [bail
bond]. x x x Otherwise, petition should be denied.

In the present case, Judge Buaya granted the ex-parte motion to grant bail on the same day
that it was filed by the accused. He did this without the required notice and hearing. He justified
his action on the ex-parte motion by arguing that the offense charged against the accused was
a bailable offense; a hearing was no longer required since bail was a matter of right. Under the
present Rules of Court, however, notice and hearing are required whether bail is a matter of
right or discretion. The Court has always stressed the indispensable nature of a bail hearing in
petitions for bail. Where bail is a matter of discretion, the grant or the denial of bail hinges on the
issue of whether or not the evidence on the guilt of the accused is strong and the determination
of whether or not the evidence is strong is a matter of judicial discretion which remains with the
judge. In order for the judge to properly exercise this discretion, he must first conduct a hearing
to determine whether the evidence of guilt is strong. This discretion lies not in the determination
of whether or not a hearing should be held, but in the appreciation and evaluation of the weight
of the prosecution’s evidence of guilt against the accused.

WHEREFORE, we find respondent Acting Presiding Judge Apolinario M. Buaya of the Regional
Trial Court, Branch 17, of Palompon, Leyte, GUILTY of Gross Ignorance of the Law and Grave
Abuse of Authority, and is hereby FINED Twenty Thousand Pesos (₱20,000.00), with a
WARNING that a repetition of the same or similar acts in the future shall merit a more serious
penalty.

GACAL VS INFANTE
Doctrine: It is axiomatic that bail cannot be allowed to a person charged with a capital offense,
or an offense punishable with reclusion perpetua or life imprisonment, without a hearing upon
notice to the Prosecution. Any judge who so allows bail is guilty of gross ignorance of the law
and the rules, and is subject to appropriate administrative sanctions.

Facts: Atty. Franklin Gacal, the private prosecutor in a criminal case entitled People v. Faustino
Ancheta, a prosecution for murder arising from the killing of Felomino O. Occasion, charges
Judge Jaime I. Infante to whose branch said criminal case was raffled for arraignment and trial,
with gross ignorance of the law, gross incompetence, and evident partiality, for the latter’s
failure to set a hearing before granting bail to the accused and for releasing him immediately
after allowing bail.

On March 18, 2003, Judge Gregorio R. Balanag, Jr. of the Municipal Circuit Trial Court of
Kiamba-Maitum, Sarangani issued a warrant for the arrest of Ancheta in connection with a
murder case. Judge Balanag did not recommend bail. Ancheta, who had meanwhile gone into
hiding, was not arrested. Upon review, the Office of the Provincial Prosecutor, acting through
Assistant Provincial Prosecutor Alfredo Barcelona, Jr., affirmed the findings and
recommendation of Judge Balanag on the offense to be charged, and accordingly filed in the
RTC an information for murder on April 21, 2003 but with a recommendation for bail in the
amount of ₱400,000.00. The case was raffled to Judge Infante’s Branch.

Judge Infante issued twin orders, one granting bail to Ancheta, and another releasing Ancheta
from custody. Atty. Gacal, upon learning of the twin orders, filed a so-called Very Urgent Motion
For Reconsideration And/Or To Cancel Bailbond With Prayer To Enforce Warrant Of Arrest Or
Issue Warrant Of Arrest Anew Or In The Alternative Very Urgent Motion For This Court To Motu
Proprio Correct An Apparent And Patent Error (very urgent motion).

In the hearing, only Atty. Gacal and his collaborating counsel appeared in court. Judge Infante
directed the public prosecutor to comment on the very urgent motion within five days from
notice, after which the motion would be submitted for resolution with or without the comment.
Ancheta, through counsel, opposed, stating that the motion did not bear the conformity of the
public prosecutor.

At the arraignment of Ancheta, the parties and their counsel appeared, but Assistant Provincial
Prosecutor Barcelona, Jr., the assigned public prosecutor, did not appear because he was then
following up his regular appointment as the Provincial Prosecutor of Sarangani Province.
Accordingly, the arraignment was reset.

On May 21, 2003, Judge Infante denied Atty. Gacal’s very urgent motion on the ground that the
motion was pro forma for not bearing the conformity of the public prosecutor, and on the further
ground that the private prosecutor had not been authorized to act as such pursuant to Section 5,
Rule 110, of the Rules of Court. Judge Infante directed that the consideration of the bail issue
be held in abeyance until after the public prosecutor had submitted a comment, because he
wanted to know the position of the public prosecutor on Atty. Gacal’s very urgent motion having
been filed without the approval of the public prosecutor.
The public prosecutor appeared, but did not file any comment. Thereupon, Atty. Gacal sought
authority to appear as a private prosecutor. The public prosecutor did not oppose Atty. Gacal’s
request. With that, Atty. Gacal moved for the reconsideration of the grant of bail to Ancheta. In
response, Judge Infante required the public prosecutor to file his comment on Atty. Gacal’s
motion for reconsideration, and again reset the arraignment of the accused. The public
prosecutor filed a comment, stating that he had recommended bail as a matter of course; that
the orders approving bail upon his recommendation and releasing the accused were proper;
and that his recommendation of bail was in effect a waiver of the public prosecutor’s right to a
bail hearing.

Atty. Gacal sought the inhibition of Judge Infante on the ground of his gross incompetence
manifested by his failure to exercise judicial power to resolve the issue of bail. Atty. Gacal
insisted that Judge Infante failed to resolve the issue of bail, although he should have acted
upon it with dispatch, because it was unusual that several persons charged with murder were
being detained while Ancheta was let free on bail even without his filing a petition for bail.

The Office of the Court Administrator (OCAd) received from the Office of the Ombudsman the
indorsement of the administrative complaint Atty. Gacal had filed against Judge Infante
forwarding the records of the administrative case for appropriate action to the Supreme Court.
Then Court Administrator Prebitero J. Velasco, Jr. required Judge Infante to comment on the
administrative complaint against him. On October 6, 2003, the OCAd received Judge Infante’s
comment dated September 22, 2003, by which he denied any transgression in the granting of
bail to Ancheta, contending that the twin orders are exactly meant as an approval of the
bailbond (property) posted by accused Ancheta, it being found to be complete and sufficient.
They are not orders granting an application for bail, as misconstrued by private prosecutor.
Further, he contends that a bail hearing was not necessary because the accused did not file an
application for bail; and because the public prosecutor had recommended bail.

Issue: Can respondent judge in granting bail to the accused dispense with the hearing of
Application for Bail?

Held: Judge Infante’s contention is unwarranted.

Even where there is no petition for bail in a case like Criminal Case No. 1138-03, a hearing
should still be held. This hearing is separate and distinct from the initial hearing to determine the
existence of probable cause, in which the trial judge ascertains whether or not there is sufficient
ground to engender a well-founded belief that a crime has been committed and that the accused
is probably guilty of the crime. The Prosecution must be given a chance to show the strength of
its evidence; otherwise, a violation of due process occurs.

The fact that the public prosecutor recommended bail for Ancheta did not warrant dispensing
with the hearing. The public prosecutor’s recommendation of bail was not material in deciding
whether to conduct the mandatory hearing or not. For one, the public prosecutor’s
recommendation, albeit persuasive, did not necessarily bind the trial judge, in whom alone the
discretion to determine whether to grant bail or not was vested. Whatever the public prosecutor
recommended, including the amount of bail, was non-binding. Nor did such recommendation
constitute a showing that the evidence of guilt was not strong. If it was otherwise, the trial judge
could become unavoidably controlled by the Prosecution.

Being the trial judge, Judge Infante had to be aware of the precedents laid down by the
Supreme Court regarding the bail hearing being mandatory and indispensable. He ought to
have remembered, then, that it was only through such hearing that he could be put in a position
to determine whether the evidence for the Prosecution was weak or strong. Hence, his
dispensing with the hearing manifested a gross ignorance of the law and the rules.

That the Prosecution did not oppose the grant of bail to Ancheta, as in fact it recommended bail,
and that the Prosecution did not want to adduce evidence were irrelevant, and did not dispense
with the bail hearing. The gravity of the charge in Criminal Case No. 1138-03 made it still
mandatory for Judge Infante to conduct a bail hearing in which he could have made on his own
searching and clarificatory questions from which to infer the strength or weakness of the
evidence of guilt. He should not have readily and easily gone along with the public prosecutor’s
opinion that the evidence of guilt, being circumstantial, was not strong enough to deny bail; else,
he might be regarded as having abdicated from a responsibility that was his alone as the trial
judge.

Ivler vs. San Pedro


G.R. No. 172716 November 17, 2010

FACTS:

Following a vehicular collision in August 2004, petitioner Jason Ivler (petitioner) was charged
before the Metropolitan Trial Court of Pasig City (MTC), with two separate offenses: (1)
Reckless Imprudence Resulting in Slight Physical Injuries for injuries sustained by respondent
Evangeline L. Ponce (respondent Ponce); and (2) Reckless Imprudence Resulting in Homicide
and Damage to Property for the death of respondent Ponce’s husband Nestor C. Ponce and
damage to the spouses Ponce’s vehicle.

Petitioner posted bail for his temporary release in both cases. On 2004, petitioner pleaded guilty
to the charge on the first delict and was meted out the penalty of public censure. Invoking this
conviction, petitioner moved to quash the Information for the second delict for placing him in
jeopardy of second punishment for the same offense of reckless imprudence.

The MTC refused quashal, finding no identity of offenses in the two cases.
The petitioner elevated the matter to the Regional Trial Court of Pasig City (RTC), in a petition
for certiorari while Ivler sought from the MTC the suspension of proceedings in criminal case,
including the arraignment his arraignment as a prejudicial question.

Without acting on petitioner’s motion, the MTC proceeded with the arraignment and, because of
petitioner’s absence, cancelled his bail and ordered his arrest.

Seven days later, the MTC issued a resolution denying petitioner’s motion to suspend
proceedings and postponing his arraignment until after his arrest. Petitioner sought
reconsideration but as of the filing of this petition, the motion remained unresolved.

ISSUES:

1. Whether petitioner forfeited his standing to seek relief from his petition for certiorari when the
MTC ordered his arrest following his non-appearance at the arraignment in Reckless
Imprudence Resulting in Slight Physical Injuries for injuries sustained by respondent; and

2. Whether petitioner’s constitutional right under the Double Jeopardy Clause bars further
proceedings in Reckless Imprudence Resulting in Homicide and Damage to Property for the
death of respondent Ponce’s husband.

RULING:

The accused negative constitutional right not to be "twice put in jeopardy of punishment for the
same offense" protects him from, among others, post-conviction prosecution for the same
offense, with the prior verdict rendered by a court of competent jurisdiction upon a valid
information.

Petitioner adopts the affirmative view, submitting that the two cases concern the same offense
of reckless imprudence. The MTC ruled otherwise, finding that Reckless Imprudence Resulting
in Slight Physical Injuries is an entirely separate offense from Reckless Imprudence Resulting in
Homicide and Damage to Property "as the [latter] requires proof of an additional fact which the
other does not."
The two charges against petitioner, arising from the same facts, were prosecuted under the
same provision of the Revised Penal Code, as amended, namely, Article 365 defining and
penalizing quasi-offenses.

The provisions contained in this article shall not be applicable. Indeed, the notion that quasi-
offenses, whether reckless or simple, are distinct species of crime, separately defined and
penalized under the framework of our penal laws, is nothing new.

The doctrine that reckless imprudence under Article 365 is a single quasi-offense by itself and
not merely a means to commit other crimes such that conviction or acquittal of such quasi-
offense bars subsequent prosecution for the same quasi-offense, regardless of its various
resulting acts, undergirded this Court’s unbroken chain of jurisprudence on double jeopardy as
applied to Article 365.

These cases uniformly barred the second prosecutions as constitutionally impermissible under
the Double Jeopardy Clause.

Our ruling today secures for the accused facing an Article 365 charge a stronger and simpler
protection of their constitutional right under the Double Jeopardy Clause. True, they are thereby
denied the beneficent effect of the favorable sentencing formula under Article 48, but any
disadvantage thus caused is more than compensated by the certainty of non-prosecution for
quasi-crime effects qualifying as "light offenses" (or, as here, for the more serious consequence
prosecuted belatedly). If it is so minded, Congress can re-craft Article 365 by extending to
quasi-crimes the sentencing formula of Article 48 so that only the most severe penalty shall be
imposed under a single prosecution of all resulting acts, whether penalized as grave, less grave
or light offenses. This will still keep intact the distinct concept of quasi-offenses. Meanwhile, the
lenient schedule of penalties under Article 365, befitting crimes occupying a lower rung of
culpability, should cushion the effect of this ruling.

Petition granted

TEODORO C. BORLONGAN, JR. vs. MAGDALENO M. PEÑA


G.R. No. 143591, May 5, 2010

Doctrine: The ruling of this Court was that posting of bail constitutes a waiver of any irregularity
in the issuance of a warrant of arrest, that has already been superseded by Section 26, Rule
114 of the Revised Rule of Criminal Procedure. The principle that the accused is precluded
from questioning the legality of the arrest after arraignment is true only if he voluntarily enters
his plea and participates during trial, without previously invoking his objections thereto.

While probable cause should first be determined before an information may be filed in court, the
prosecutor is not mandated to require the respondent to submit his counter-affidavits to oppose
the complaint. In the determination of probable cause, the prosecutor may solely rely on the
complaint, affidavits and other supporting documents submitted by the complainant. If he does
not find probable cause, the prosecutor may dismiss outright the complaint or if he finds
probable cause or sufficient reason to proceed with the case, he shall issue a resolution and file
the corresponding information.

Facts: Atty. Magdaleno M. Peña instituted a civil case for recovery of agent’s compensation and
expenses, damages, and attorney’s fees against Urban Bank and herein petitioners, before the
Regional Trial Court (RTC) of Negros Occidental, Bago City. Atty. Peña anchored his claim for
compensation on the Contract of Agency allegedly entered into with the petitioners, wherein the
former undertook to perform such acts necessary to prevent any intruder and squatter from
unlawfully occupying Urban Bank’s property located along Roxas Boulevard, Pasay City.

Petitioners filed a Motion to Dismiss arguing that they never appointed the respondent as agent
or counsel. Documents were presented in an attempt to show that the respondent was
appointed as agent by ISCI and not by Urban Bank or by the petitioners.

Atty. Peña claimed that said documents were falsified because the alleged signatories did not
actually affix their signatures, and the signatories were neither stockholders nor officers and
employees of ISCI.

The City Prosecutor found probable cause for the indictment of petitioners for four (4) counts of
the crime of Introducing Falsified Documents, penalized by the second paragraph of Article 172
of the Revised Penal Code. The City Prosecutor concluded that the documents were falsified
because the alleged signatories untruthfully stated that ISCI was the principal of the respondent;
that petitioners knew that the documents were falsified considering that the signatories were
mere dummies; and that the documents formed part of the record of Civil Case No. 754 where
they were used by petitioners as evidence in support of their motion to dismiss, and then
adopted in their answer and in their Pre-Trial Brief. Subsequently, the corresponding
Informations were filed and thereafter warrants were issued.

Petitioners filed an Omnibus Motion to Quash, Recall Warrants of Arrest and/or For
Reinvestigation. Petitioners insisted that they were denied due process because of the non-
observance of the proper procedure on preliminary investigation prescribed in the Rules of
Court. Specifically, they claimed that they were not afforded the right to submit their counter-
affidavit. Then they argued that since no such counter-affidavit and supporting documents were
submitted by the petitioners, the trial judge merely relied on the complaint-affidavit and
attachments of the respondent in issuing the warrants of arrest, also in contravention with the
Rules of Court. Petitioners further prayed that the information be quashed for lack of probable
cause. Moreover, one of the accused, i.e., Ben Lim, Jr., is not even a director of Urban Bank,
contrary to what complainant stated. Lastly, petitioners posited that the criminal cases should
have been suspended on the ground that the issue being threshed out in the civil case is a
prejudicial question.

The MTCC denied the omnibus motion primarily on the ground that preliminary investigation
was not available in the instant case – which fell within the jurisdiction of the first-level court.
The court, likewise, upheld the validity of the warrant of arrest, saying that it was issued in
accordance with the Rules of Court. Besides, the court added, petitioners could no longer
question the validity of the warrant since they already posted bail. The court also believed that
the issue involved in the civil case was not a prejudicial question, and, thus, denied the prayer
for suspension of the criminal proceedings. Lastly, the court was convinced that the
Informations contained all the facts necessary to constitute an offense.

Petitioners immediately instituted a special civil action for Certiorari and Prohibition with Prayer
for Writ of Preliminary Injunction and Temporary Restraining Order (TRO) before the Court of
Appeals. They, likewise, questioned the court’s conclusion that by posting bail, petitioners
already waived their right to assail the validity of the warrants of arrest. The Court of Appeals
dismissed the petition

Issue: Whether or not the Court of Appeals gravely abuse its discretion in denying the
Petitioners’ motion for reinvestigation and recall of the warrants of arrest.

Ruling: Yes. The ruling of this Court was that posting of bail constitutes a waiver of any
irregularity in the issuance of a warrant of arrest, that has already been superseded by Section
26, Rule 114 of the Revised Rule of Criminal Procedure. The principle that the accused is
precluded from questioning the legality of the arrest after arraignment is true only if he
voluntarily enters his plea and participates during trial, without previously invoking his objections
thereto. Herein petitioners filed the Omnibus Motion to Quash, Recall Warrants of Arrest and/or
For Reinvestigation on the same day that they posted bail. Their bail bonds likewise expressly
contained a stipulation that they were not waiving their right to question the validity of their
arrest. On the date of their arraignment, petitioners refused to enter their plea due to the fact
that the issue on the legality of their arrest is still pending with the Court. Thus, when the court a
quo entered a plea of not guilty for them, there was no valid waiver of their right to preclude
them from raising the same with the Court of Appeals or this Court. The posting of bail bond
was a matter of imperative necessity to avert their incarceration; it should not be deemed as a
waiver of their right to assail their arrest.

On the second point, the crime to which petitioners were charged was defined and penalized
under second paragraph of Article 172 in relation to Article 171 of the Revised Penal Code.
Art. 172. Falsification by private individual and use of falsified documents. — The penalty of
prision correccional in its medium and maximum periods and a fine of not more than ₱5,000
pesos shall be imposed upon:

1. Any private individual who shall commit any of the falsifications enumerated in the next
preceding article in any public or official document or letter of exchange or any other kind of
commercial document; and

2. Any person who, to the damage of a third party, or with the intent to cause such damage,
shall in any private document commit any of the acts of falsification enumerated in the next
preceding article.

Any person who shall knowingly introduce in evidence in any judicial proceeding or to the
damage of another or who, with the intent to cause such damage, shall use any of the false
documents embraced in the next preceding article or in any of the foregoing subdivisions of this
article, shall be punished by the penalty next lower in degree.

Prision correccional in its medium and maximum periods translates to imprisonment of 2 years,
4 months and 1 day. The next lower in degree to prision correccional is arresto mayor in its
maximum period to prision correccional in its minimum period which translates to 4 months and
1 day to 2 years and 4 months of imprisonment. Since the crime committed is not covered by
the Rules of Summary Procedure, the case falls within the exclusive jurisdiction of the first level
courts but applying the ordinary rules. In such instance, preliminary investigation as defined in
Section 1, Rule 112 of the 1985 Rules of Criminal Procedure is not applicable since such
section covers only crimes cognizable by the RTC. That which is stated in Section 9(a) is the
applicable rule.

Under this Rule, while probable cause should first be determined before an information may be
filed in court, the prosecutor is not mandated to require the respondent to submit his counter-
affidavits to oppose the complaint. In the determination of probable cause, the prosecutor may
solely rely on the complaint, affidavits and other supporting documents submitted by the
complainant. If he does not find probable cause, the prosecutor may dismiss outright the
complaint or if he finds probable cause or sufficient reason to proceed with the case, he shall
issue a resolution and file the corresponding information.

This Court, however, cannot find these documents sufficient to support the existence of
probable cause.

We do not see how it can be concluded that the documents mentioned by respondent in his
complaint-affidavit were falsified. In his complaint, Atty. Peña stated that Herman Ponce, Julie
Abad and Marilyn Ong, the alleged signatories of the questioned letters, did not actually affix
their signatures therein; and that they were not actually officers or stockholders of ISCI. He
further claimed that Enrique Montilla’s signature appearing in another memorandum addressed
to respondent was forged. These averments are mere assertions which are insufficient to
warrant the filing of the complaint or worse the issuance of warrants of arrest. These averments
cannot be considered as proceeding from the personal knowledge of herein respondent who
failed to, basically, allege that he was present at the time of the execution of the documents.
Neither was there any mention in the complaint-affidavit that herein respondent was familiar with
the signatures of the mentioned signatories to be able to conclude that they were forged. What
Atty. Peña actually stated were but sweeping assertions that the signatories are mere dummies
of ISCI and that they are not in fact officers, stockholders or representatives of the corporation.
Again, there is no indication that the assertion was based on the personal knowledge of the
affiant.

The reason for the requirement that affidavits must be based on personal knowledge is to guard
against hearsay evidence. A witness, therefore, may not testify as what he merely learned from
others either because he was told or read or heard the same. Such testimony is considered
hearsay and may not be received as proof of the truth of what he has learned. Hearsay is not
limited to oral testimony or statements; the general rule that excludes hearsay as evidence
applies to written, as well as oral statements.

The requirement of personal knowledge should have been strictly applied considering that
herein petitioners were not given the opportunity to rebut the complainant’s allegation through
counter-affidavits.

Lastly, what tainted the procedure further was that the Judge issued a warrant for the arrest of
the petitioners, including, Mr. Ben Lim, Jr. despite the filing of the Omnibus Motion to Quash,
Recall Warrants of Arrest and/or For Reinvestigation raising among others the issue that Mr.
Ben Lim, Jr., was not even a member of the board of directors. With the filing of the motion, the
judge is put on alert that an innocent person may have been included in the complaint.

The Court cannot accept as mere oversight the mistake of respondent judge since it was at the
expense of liberty. This cannot be condoned.

In the issuance of a warrant of arrest, the mandate of the Constitution is for the judge to
personally determine the existence of probable cause:

Section 2, Article III of the Constitution provides:

Section 2. The right of the people to be secure in their persons, houses, papers and effects
against unreasonable searches and seizures of whatever nature and for any purpose shall be
inviolable, and no search warrant or warrant of arrest shall issue except upon probable cause to
be determined personally by the judge after examination under oath or affirmation of the
complainant and the witnesses he may produce, and particularly describing the place to be
searched and the persons or things to be seized.

Corollary thereto, Section 9(b) of the 1985 Rules of Criminal Procedure provides:

Sec. 9. Cases not falling under the original jurisdiction of the Regional Trial Courts nor covered
by the Rule on Summary Procedure.

(a) x x x.
(b) Where filed directly with the Municipal Trial Court. — If the complaint or information is filed
directly with the Municipal Trial Court, the procedure provided for in Section 3(a) of this Rule
shall likewise be observed. If the judge finds no sufficient ground to hold the respondent for trial,
he shall dismiss the complaint or information. Otherwise, he shall issue a warrant of arrest after
personally examining in writing and under oath the complainant and his witnesses in the form of
searching questions and answers.

Enshrined in our Constitution is the rule that "[n]o x x x warrant of arrest shall issue except upon
probable cause to be determined personally by the judge after examination under oath or
affirmation of the complainant and the witnesses he may produce, and particularly describing x
x x the persons x x x to be seized." Interpreting the words "personal determination," we said in
Soliven v. Makasiar that it does not thereby mean that judges are obliged to conduct the
personal examination of the complainant and his witnesses themselves. To require thus would
be to unduly laden them with preliminary examinations and investigations of criminal complaints
instead of concentrating on hearing and deciding cases filed before them. Rather, what is
emphasized merely is the exclusive and personal responsibility of the issuing judge to satisfy
himself as to the existence of probable cause. To this end, he may: (a) personally evaluate the
report and the supporting documents submitted by the prosecutor regarding the existence of
probable cause and, on the basis thereof, issue a warrant of arrest; or (b) if on the basis thereof
he finds no probable cause, disregard the prosecutor's report and require the submission of
supporting affidavits of witnesses to aid him in determining its existence. What he is never
allowed to do is to follow blindly the prosecutor's bare certification as to the existence of
probable cause. Much more is required by the constitutional provision. Judges have to go over
the report, the affidavits, the transcript of stenographic notes if any, and other documents
supporting the prosecutor's certification. Although the extent of the judge's personal examination
depends on the circumstances of each case, to be sure, he cannot just rely on the bare
certification alone but must go beyond it. This is because the warrant of arrest issues not on the
strength of the certification standing alone but because of the records which sustain it. He
should even call for the complainant and the witnesses to answer the court's probing questions
when the circumstances warrant.

An arrest without a probable cause is an unreasonable seizure of a person, and violates the
privacy of persons which ought not to be intruded by the State.

Measured against the constitutional mandate and established rulings, there was here a clear
abdication of the judicial function and a clear indication that the judge blindly followed the
certification of a city prosecutor as to the existence of probable cause for the issuance of a
warrant of arrest with respect to all of the petitioners. The careless inclusion of Mr. Ben Lim, Jr.,
in the warrant of arrest gives flesh to the bone of contention of petitioners that the instant case is
a matter of persecution rather than prosecution. On this ground, this Court may enjoin the
criminal cases against petitioners. As a general rule, criminal prosecutions cannot be enjoined.
However, there are recognized exceptions which, as summarized in Brocka v. Enrile, are:

a. To afford adequate protection to the constitutional rights of the accused;


b. When necessary for the orderly administration of justice or to avoid oppression or multiplicity
of actions;

c. When there is a prejudicial question which is sub judice;

d. When the acts of the officer are without or in excess of authority;

e. Where the prosecution is under an invalid law, ordinance or regulation;

f. When double jeopardy is clearly apparent;

g. Where the court had no jurisdiction over the offense;

h. Where it is a case of persecution rather than prosecution;

i. Where the charges are manifestly false and motivated by the lust for vengeance; and

j. When there is clearly no prima facie case against the accused and a motion to quash on that
ground has been denied.
Miranda vs. Tulucio

DOCTRINES:

1. Custody of the law is required before the court can act upon the application for bail, but is
not required for the adjudication of other reliefs sought by the defendant where the mere
application therefor constitutes a waiver of the defense of lack of jurisdiction over the person
of the accused. Custody of the law is accomplished either by arrest or voluntary surrender,
while jurisdiction over the person of the accused is acquired upon his arrest or voluntary
appearance. One can be under the custody of the law but not yet subject to the jurisdiction
of the court over his person, such as when a person arrested by virtue of a warrant files a
motion before arraignment to quash the warrant. On the other hand, one can be subject to
the jurisdiction of the court over his person, and yet not be in the custody of the law, such as
when an accused escapes custody after his trial has commenced. Being in the custody of
the law signifies restraint on the person, who is thereby deprived of his own will and liberty,
binding him to become obedient to the will of the law. Custody of the law is literally custody
over the body of the accused. It includes, but is not limited to, detention.

2. The purpose of bail is to secure one’s release and it would be incongruous to grant bail to
one who is free. Bail is the security required and given for the release of a person who is in
the custody of law.’" The rationale behind this special rule on bail is that it discourages and
prevents resort to the former pernicious practice wherein the accused could just send
another in his stead to post his bail, without recognizing the jurisdiction of the court by his
personal appearance therein and compliance with the requirements therefor.

3. There is, however, an exception to the rule that filing pleadings seeking affirmative relief
constitutes voluntary appearance, and the consequent submission of one’s person to the
jurisdiction of the court. This is in the case of pleadings whose prayer is precisely for the
avoidance of the jurisdiction of the court, which only leads to a special appearance. These
pleadings are: (1) in civil cases, motions to dismiss on the ground of lack of jurisdiction over
the person of the defendant, whether or not other grounds for dismissal are included; (2) in
criminal cases, motions to quash a complaint on the ground of lack of jurisdiction over the
person of the accused; and (3) motions to quash a warrant of arrest.

FACTS:

Two burnt cadavers were discovered and were later identified as the dead bodies of Vicente
Bauzon and Elizer Tuliao, son respondent Virgilio Tuliao who is now under the witness
protection program. Informations for murder were filed against SPO1 Wilfredo Leaño, SPO1
Ferdinand Marzan, SPO1 Ruben B. Agustin, SPO2 Alexander Micu, SPO2 Rodel Maderal, and
SPO4 Emilio Ramirez in the RTC. The venue was later transferred to Manila. The RTC of
Manila convicted all of the accused and sentenced them to two counts of reclusion perpetua
except SPO2 Maderal who was yet to be arraigned at that time, being at large. The case was
appealed to the Supreme Court on automatic review where the accused therein are acquitted
on the ground of reasonable doubt.

SPO2 Maderal was arrested. He executed a sworn confession and identified petitioners Jose C.
Miranda, PO3 Romeo B. Ocon, and SPO3 Alberto P. Dalmacio, a certain Boyet dela Cruz and
Amado Doe, as the persons responsible for the deaths of Vicente Bauzon and Elizer Tuliao.

Tuliao filed a criminal complaint for murder against petitioners, Boyet dela Cruz, and Amado
Doe, and submitted the sworn confession of SPO2 Maderal. Judge Wilfredo Tumaliuan issued
warrants of arrest against petitioners and SPO2 Maderal. Petitioners filed an urgent motion to
complete preliminary investigation, to reinvestigate, and to recall and/or quash the warrants of
arrest. Judge Tumaliuan noted the absence of petitioners and issued a Joint Order denying said
urgent motion on the ground that, since the court did not acquire jurisdiction over their persons,
the motion cannot be properly heard by the court. the new Presiding Judge Anastacio D.
Anghad took over the case and issued a Joint Order reversing the Joint Order of Judge
Tumaliuan. Consequently, he ordered the cancellation of the warrant of arrest issued against
petitioner Miranda. He likewise applied this Order to petitioners Ocon and Dalmacio.

Tuliao filed a petition for certiorari, mandamus and prohibition with the Supreme Court, with
prayer for a Temporary Restraining Order, seeking to enjoin Judge Anghad from further
proceeding with the case, and seeking to nullify the Orders and Joint Orders of Judge Anghad.
The SC issued a Resolution resolving to grant the prayer for a temporary restraining order
against Judge Anghad from further proceeding with the criminal cases and referred the said
motion to the Court of Appeals in view of the previous referral to it of respondent’s petition for
certiorari, prohibition and mandamus. Shortly after the aforesaid resolution, Judge Anghad
issued a Joint Order dismissing the two Informations for murder against petitioners.

The Court of Appeals rendered the assailed decision granting the petition and ordering the
reinstatement of the criminal cases in the RTC, as well as the issuance of warrants of arrest
against petitioners and SPO2 Maderal.

ISSUE:
Whether or not an application for admission to bail requires the accused to be in the custody of
law or otherwise deprived of his liberty

RULING: YES. A person applying for admission to bail must be in the custody of the law
or otherwise deprived of his liberty.

A person who has not submitted himself to the jurisdiction of the court has no right to invoke the
processes of that court. Respondent Judge should have diligently ascertained the whereabouts
of the applicant and that he indeed had jurisdiction over the body of the accused before
considering the application for bail.

Custody of the law is required before the court can act upon the application for bail, but is not
required for the adjudication of other reliefs sought by the defendant where the mere application
therefor constitutes a waiver of the defense of lack of jurisdiction over the person of the
accused. Custody of the law is accomplished either by arrest or voluntary surrender, while
jurisdiction over the person of the accused is acquired upon his arrest or voluntary appearance.
One can be under the custody of the law but not yet subject to the jurisdiction of the court over
his person, such as when a person arrested by virtue of a warrant files a motion before
arraignment to quash the warrant. On the other hand, one can be subject to the jurisdiction of
the court over his person, and yet not be in the custody of the law, such as when an accused
escapes custody after his trial has commenced. Being in the custody of the law signifies
restraint on the person, who is thereby deprived of his own will and liberty, binding him to
become obedient to the will of the law. Custody of the law is literally custody over the body of
the accused. It includes, but is not limited to, detention.

The purpose of bail is to secure one’s release and it would be incongruous to grant bail to one
who is free. Bail is the security required and given for the release of a person who is in the
custody of law.’" The rationale behind this special rule on bail is that it discourages and prevents
resort to the former pernicious practice wherein the accused could just send another in his stead
to post his bail, without recognizing the jurisdiction of the court by his personal appearance
therein and compliance with the requirements therefor.

There is, however, an exception to the rule that filing pleadings seeking affirmative relief
constitutes voluntary appearance, and the consequent submission of one’s person to the
jurisdiction of the court. This is in the case of pleadings whose prayer is precisely for the
avoidance of the jurisdiction of the court, which only leads to a special appearance. These
pleadings are: (1) in civil cases, motions to dismiss on the ground of lack of jurisdiction over the
person of the defendant, whether or not other grounds for dismissal are included; (2) in criminal
cases, motions to quash a complaint on the ground of lack of jurisdiction over the person of the
accused; and (3) motions to quash a warrant of arrest.

The circumstances forcing us to require custody of the law in applications for bail are not
present in motions to quash the warrant of arrest. If we allow the granting of bail to persons not
in the custody of the law, it is foreseeable that many persons who can afford the bail will remain
at large, and could elude being held to answer for the commission of the offense if ever he is
proven guilty. On the other hand, if we allow the quashal of warrants of arrest to persons not in
the custody of the law, it would be very rare that a person not genuinely entitled to liberty would
remain scot-free. This is because it is the same judge who issued the warrant of arrest who will
decide whether or not he followed the Constitution in his determination of probable cause, and
he can easily deny the motion to quash if he really did find probable cause after personally
examining the records of the case.

Moreover, pursuant to the presumption of regularity of official functions, the warrant continues in
force and effect until it is quashed and therefore can still be enforced on any day and at any
time of the day and night. Furthermore, the continued absence of the accused can be taken
against him in the determination of probable cause, since flight is indicative of guilt.
DIOSDADO JOSE ALLADO vs. HON. ROBERTO C. DIOKNO
G.R. No. 113630 May 5, 1994

DOCTRINE: Section 2, Art. III, of the 1987 Constitution, lays down the requirements for the
issuance of a warrant of arrest, i.e., a warrant of arrest shall issue only upon probable cause to
be determined personally by the judge after examination under oath or affirmation of the
complainant and the witnesses he may produce.

Probable cause for an arrest or for the issuance of a warrant of arrest has been defined as such
facts and circumstances which would lead a reasonable discreet and prudent man to believe
that an offense has been committed by the person sought to be arrested.  And as a protection
against false prosecution and arrest, it is the knowledge of facts, actual or apparent, strong
enough to justify a reasonable man in the belief that he was lawful grounds for arresting the
accused.

FACTS: Petitioners Diosdado Allado and Roberto Mendoza, alumni of the College of Law,
University of the Philippines, are partners of the Law Firm of Salonga, Hernandez and Allado. In
the practice of their profession, and on the basis of an alleged extrajudicial confession of a
security guard, they have been accused of the heinous crime of kidnapping with murder by the
Presidential Anti-Crime Commission (PACC) and ordered arrested without bail by respondent
judge.

The focal source of the information against petitioners is the sworn statement of Security Guard
Umbal, a discharge of the Phil. Constabulary, implicating them as the brains behind the alleged
kidnapping and slaying of one Eugen Alexander Van Twest, a German national. In that
extrajudicial confession, Umbal claimed that he and his companions were met by petitioners at
Silahis Hotel and in exchange for P2.5M the former undertook to apprehend Van Twest who
allegedly had an international warrant of arrest against him. A day after Umbal executed his
extrajudicial confession, the operatives of the PACC, armed with a search warrant issued by
Judge Barrios, raided the two (2) dwellings of Santiago. The raiders recovered a blue Nissan
Pathfinder and assorted firearms and ammunition and placed Santiago and his trusted aide,
Efren Madolid, under arrest. Also arrested later that day were Antonio and Bato who were found
to have in their possession several firearms and ammunition and Van Twest's Cartier
sunglasses.

After evaluating of the evidence, the case was referred to the DOJ for the institution of criminal
proceedings. Thereafter, Senior State Prosecutor Ferdinand prosecutor Ferdinand R. Abesamis
issued a subpoena to petitioners informing them that a complaint
was filed against them by PACC TF-Habagat, directing them to appear on
30 September 1993 at the Multi-Purpose Hall of the Department of Justice and to submit their
counter-affidavits. Attached to the subpoena were copies of the affidavits executed by Umbal
and members of the team who raided the two (2) dwellings of Santiago.

The Petitioners maintain that the records of the preliminary investigation which respondent
judge solely relied upon failed to establish probable cause against them to justify the issuance
of the warrant of arrest. Petitioners likewise assail the prosecutors' "clear sign of bias and
impartiality. On the other hand, the Office of the Solicitor General argues that the determination
of probable cause is a function of the judge who is merely required to personally appreciate
certain facts to convince him that the accused probably committed the crime charged.
ISSUE: Whether the respondent judge failed to establish probable cause against them to justify
the issuance of the warrant of arrest?

RULING: No. Accordingly, before issuing a warrant of arrest, the judge must satisfy himself that
based on the evidence submitted there is sufficient proof that a crime has been committed and
that the person to be arrested is probably guilty thereof. In the Order of respondent judge dated
11 February 1994, it is expressly stated that "[t]his court after careful evaluation of the evidence
on record, believes and rules that probable cause exists; and therefore, a warrant of arrest
should be issued." However, we are unable to see how respondent judge arrived at such ruling.
We have painstakingly examined the records and we cannot find any support for his
conclusion. 

The Presidential Anti-Crime Commission relies heavily on the sworn statement of Security
Guard Umbal who supposedly confessed his participation in the alleged kidnapping and murder
of Van Twest. For one, there is serious doubt on Van Twest's reported death since the corpus
delicti has not been established, nor have his remains been recovered. Umbal claims that Van
Twest was completely burned into ashes with the use of gasoline and rubber tires from around
ten o'clock in the evening to six o'clock the next morning. This is highly improbable, if not
ridiculous. A human body cannot be pulverized into ashes by simply burning it with the use of
gasoline and rubber tires in an open field. Even crematoria use entirely closed incinerators
where the corpse is subjected to intense heat. Thereafter, the remains undergo a process
where the bones are completely ground to dust. Then, the extrajudicial statement of Umbal
suffers from material inconsistencies.
Verily, respondent judge committed grave abuse of discretion in issuing the warrant for the
arrest of petitioners it appearing that he did not personally examine the evidence nor did he call
for the complainant and his witnesses in the face of their incredible accounts. Instead, he merely
relied on the certification of the prosecutors that probable cause existed. For, otherwise, he
would have found out that the evidence thus far presented was utterly insufficient to warrant the
arrest of petitioners. In this regard, we restate the procedure we outlined in various cases we
have already decided.
In Soliven v. Makasiar,  we said that the judge (a) shall personally evaluate the report and the
supporting documents submitted by the fiscal regarding the existence of probable cause and, on
the basis thereof, issue a warrant of arrest; or, (b) if on the basis thereof he finds no probable
cause, may disregard the fiscal's report and require the submission of supporting affidavits of
witnesses to aid him in arriving at a conclusion on the existence of probable cause.
Clearly, probable cause may not be established simply by showing that a trial judge subjectively
believes that he has good grounds for his action. Good faith is not enough. If subjective good
faith alone were the test, the constitutional protection would be demeaned and the people would
be "secure in their persons, houses, papers and effects" only in the fallible discretion of the
judge. Thus, it appears in the instant case that the prosecutors have similarly misappropriated, if
not abused, their discretion. If they really believed that petitioners were probably guilty, they
should have armed themselves with facts and circumstances in support of that belief; for mere
belief is not enough. They should have presented sufficient and credible evidence to
demonstrate the existence of probable cause.
In the case at bench, the undue haste in the filing of the information and the inordinate interest
of the government cannot be ignored. Hence, the warrant of arrest issued against them is SET
ASIDE and respondent Judge Roberto C. Diokno is ENJOINED from proceeding any further
against herein petitioners in Crim. Case No. 94-1757 of the Regional Trial Court of Makati.

G.R. No. 113930 March 5, 1996


PAUL G. ROBERTS, JR. vs. THE COURT OF APPEALS

DOCTRINE: Instances when judges are required and when not required to personally examine
in writing and under oath the complainant and the witnesses before issuing warrants of arrest.
As to the first, a warrant can issue only if the judge is satisfied after an examination in writing
and under oath of the complainant and the witnesses, in the form of searching questions and
answers, that a probable cause exists and that there is a necessity of placing the respondent
under immediate custody in order not to frustrate the ends of justice. As to the second, this
Court held in Soliven vs. Makasiar that the judge is not required to personally examine the
complainant and the witnesses, but following established doctrine and procedure, he shall: (1)
personally evaluate the report and supporting documents submitted by the fiscal regarding the
existence of probable cause and, on the basis thereof, issue a warrant of arrest; or (2) if on the
basis thereof he finds no probable cause, he may disregard the fiscal’s report and require the
submission of supporting affidavits of witnesses to aid him in arriving at a conclusion as to the
existence of probable cause.

FACTS: Pepsi Cola Product Phils., Inc. has a Number Fever Promotion where “all holders of
crowns and/or caps of Pepsi products bearing the winning 3-digit number will win the prize
printed on the crown/cap. On May 25, 1992, it was announced that the winning number for the
next day was “349”. Several thousand holders of “349” went then to Pepsi to redeem but Pepsi
refused to pay. These holders filed for estafa against the officers of Pepsi.

After several procedural maneuvers by petitioners’ counsel, including a petition for review with
the Secretary of Justice on the finding of probable cause by the prosecutor, respondent Judge
denied the Motion to Suspend Proceedings and to Hold in Abeyance Issuance of Warrants of
Arrest and the Motion to Defer arraignment, and directed the issuance of the warrants of arrest
and settling the arraignment.

Petitioner filed a certiorari with the CA with application for TRO against Judge Asuncion alleging
grave abuse of discretion. CA granted the TRO, however, with the issuance of the decision of
the Secretary of Justice dismissing the petition for review, the CA dismissed the case as moot.
The motion for reconsideration was denied.

Petitioners filed the instant petition that Judge Asuncion committed grave abuse of discretion in
ordering the issuance of warrants of arrest without examining the records of the preliminary
investigation.
ISSUE: Whether or not respondent judge acted with grave abuse of discretion in issuing the
warrants of arrest without examination of preliminary investigation records.

HELD: Yes, Judge Asuncion acted with grave abuse of discretion in issuing the warrants of
arrest. Under the existing laws, the instances when judges are required and when not required
to personally examine in writing and under oath the complainant and the witnesses before
issuing warrants of arrest. As to the first, a warrant can issue only if the judge is satisfied after
an examination in writing and under oath of the complainant and the witnesses, in the form of
searching questions and answers, that a probable cause exists and that there is a necessity of
placing the respondent under immediate custody in order not to frustrate the ends of justice. As
to the second, this Court held in Soliven vs. Makasiar that the judge is not required to personally
examine the complainant and the witnesses, but following established doctrine and procedure,
he shall: (1) personally evaluate the report and supporting documents submitted by the fiscal
regarding the existence of probable cause and, on the basis thereof, issue a warrant of arrest;
or (2) if on the basis thereof he finds no probable cause, he may disregard the fiscal’s report
and require the submission of supporting affidavits of witnesses to aid him in arriving at a
conclusion as to the existence of probable cause.

Sound policy supports this procedure, “otherwise judges would be unduly laden with the
preliminary examination and investigation of criminal complaints instead of concentrating on
hearing and deciding cases filed before their courts.” It must be emphasized that judges must
not rely solely on the report or resolution of the fiscal (now prosecutor); they must evaluate the
report and the supporting documents.

First, the determination of probable cause is a function of the Judge. It is not for the Provincial
Fiscal or Prosecutor nor the Election Supervisor to ascertain. Only the Judge and the Judge
alone makes this determination.
Second, the preliminary inquiry made by a Prosecutor does not bind the Judge. It merely assists
him to make the determination of probable cause. The Judge does not have to follow what the
Prosecutor presents to him. By itself, the Prosecutor’s certification of probable cause is
ineffectual. It is the report, the affidavits, the transcripts of stenographic notes (if any), and all
other supporting documents behind the Prosecutor’s certification which are material in assisting
the Judge to make his determination.
Unfortunately, in this case, no affidavits of the witnesses, transcripts of stenographic notes of
the proceedings during the preliminary investigation, or other documents submitted in the
course thereof were found in the records. Clearly, when respondent Judge Asuncion issued the
assailed order of 17 May 1993 directing, among other things, the issuance of warrants of arrest,
he had only the information, amended information, and Joint Resolution as bases thereof. He
did not have the records or evidence supporting the prosecutor’s finding of probable cause. And
strangely enough, he made no specific finding of probable cause; he merely directed the
issuance of warrants of arrest “after June 21, 1993.” It may, however, be argued that the
directive presupposes a finding of probable cause. But then compliance with a constitutional
requirement for the protection of individual liberty cannot be left to presupposition, conjecture, or
even convincing logic.

PANFILO M. LACSON v. THE EXECUTIVE SECRETARY, THE SANDIGANBAYAN,


OFFICE OF THE SPECIAL PROSECUTOR, THE DEPARTMENT OF JUSTICE, MYRNA
ABALORA, NENITA ALAP-AP, IMELDA PANCHO MONTERO, and THE PEOPLE OF THE
PHILIPPINES

G.R. No. 128096 January 20, 1999

DOCTRINE:
“… an offense is said to have been committed in relation to the office if it (the offense) is
"intimately connected" with the office of the offender and perpetrated while he was in the
performance of his official functions. This intimate relation between the offense charged and the
discharge of official duties "must be alleged in the informations."
As to how the offense charged be stated in the informations, Section 9, Rule 110 of the Revised
Rules of Court mandates:
Sec. 9 Couse of accusation — The acts or omissions complied of as constituting the
offense must be stated in ordinary and concise language without repetition not necessarily in
the terms of the statute defining the offense, but in such from as is sufficient to enable a person
of common understanding to know what offense is intended to be charged, and enable the court
to pronounce proper judgment. (Emphasis supplied)
FACTS:
In the early morning of May 18, 1995, eleven (11) persons believed to be members of
the Kuratong Baleleng gang, reportedly an organized crime syndicate which had been involved
in a spate of bank robberies in Metro Manila, where slain along Commonwealth Avenue in
Quezon City by elements of the Anti-Bank Robbery and Intelligence Task Group (ABRITG)
headed by Chief Superintendent Jewel Canson of the PNP, and the petitioner and intervenors
as among the members.
Acting on a media expose of SPO2 Eduardo delos Reyes, a member of the CIC, that what
actually transpired at dawn of May 18, 1995 was a summary execution (or a rub out) and not a
shoot-out between the Kuratong Baleleng gang members and the ABRITG. Ombudsman
Aniano Desierto investigated the case which led to the filing of eleven (11) informations for
multiple murder with the Sandiganbayan.
On March 5-6, 1996, all the accused filed separate motions questioning the jurisdiction of the
Sandiganbayan, asserting that under the amended informations, the cases fall within the
jurisdiction of the Regional Trial Court.
The Sandiganbayan admitted the amended information and ordered the cases transferred to the
Quezon City Regional Trial Court which the Office of the Special Prosecutor moved for a
reconsideration, insisting that the cases should remain with the Sandiganbayan.
Subsequently, the Sandiganbayan promulgated a Resolution denying the motion for
reconsideration of the Special Prosecutor, ruling that it "stands pat in its resolution…” On the
same day, however, the Sandiganbayan issued and ADDENDUM to its Resolution, stating that
the said Court has competence to take cognizance of these cases. The multiple murder charge
against petitioner and intervenors falls under Section 4 [paragraph b] of R.A. 8249. Section 4
requires that the offense charged must be committed by the offender in relation to his office in
order for the Sandiganbayan to have jurisdiction over it. This jurisdictional requirement is in
accordance with Section 5, Article XIII of the 1973 Constitution which mandated that the
Sandiganbayan shall have jurisdiction over criminal cases committed by the public officers and
employees, including those in goverment-owned or controlled corporations, "in relation to their
office as may be determined by law."
The amended informations however, appears to be wanting of specific factual averments to
show the intimate relation/connection between the offense charged and the discharge of official
function of the offenders.
ISSUE:

Did the amended informations successfully establish the intimate connection between
the offense charged and the discharge of official functions of the PNP officers, which is an
essential factor to determine whether the case falls within the Sandiganbayan’s or Regional
Trial Court’s jurisdiction?

RULING:
No, the failure to establish in the amended informations that the charge of murder was
intimately connected with the discharge of official functions of the accused PNP officers, the
offense charged in the subject criminal cases is plain murder and, therefore, within the exclusive
original jurisdiction of the Regional Trial Court, not the Sandiganbayan.
In People vs. Montejo, it was held that an offense is said to have been committed in relation to
the office if it (the offense) is "intimately connected" with the office of the offender and
perpetrated while he was in the performance of his official functions. This intimate relation
between the offense charged and the discharge of official duties "must be alleged in the
informations."
As to how the offense charged be stated in the informations, Section 9, Rule 110 of the Revised
Rules of Court mandates:
Sec. 9 Couse of accusation — The acts or omissions complied of as constituting the
offense must be stated in ordinary and concise language without repetition not necessarily in
the terms of the statute defining the offense, but in such from as is sufficient to enable a person
of common understanding to know what offense is intended to be charged, and enable the court
to pronounce proper judgment. (Emphasis supplied)
As early as 1954, the Supreme Court pronounced that "the factor that characterizes the charge
is the actual recital of the facts." The real nature of the criminal charge is determined not from
the caption or preamble of the informations nor from the specification of the provision of law
alleged to have been violated, they being conclusions of law, but by the actual recital of facts in
the complaint or information.
Applying these, the Court finds the amended informations for murder against herein petitioner
and intervenors wanting of specific factual averments to show the intimate relation/connection
between the offense charged and the discharge of official function of the accused PNP officers.
The mere allegation in the amended information that the offense was committed by the accused
public officer in relation to his office is not sufficient.

G.R. No. 128096 January 20, 1999

DOCTRINE:
“… an offense is said to have been committed in relation to the office if it (the offense) is
"intimately connected" with the office of the offender and perpetrated while he was in the
performance of his official functions. This intimate relation between the offense charged
and the discharge of official duties "must be alleged in the informations."
As to how the offense charged be stated in the informations, Section 9, Rule 110 of the
Revised Rules of Court mandates:
Sec. 9 Couse of accusation — The acts or omissions complied of as constituting the
offense must be stated in ordinary and concise language without repetition not
necessarily in the terms of the statute defining the offense, but in such from as is
sufficient to enable a person of common understanding to know what offense is intended
to be charged, and enable the court to pronounce proper judgment. (Emphasis supplied)
FACTS:

In the early morning of May 18, 1995, eleven (11) persons believed to be members
of the Kuratong Baleleng gang, reportedly an organized crime syndicate which had been
involved in a spate of bank robberies in Metro Manila, where slain along Commonwealth
Avenue in Quezon City by elements of the Anti-Bank Robbery and Intelligence Task
Group (ABRITG) headed by Chief Superintendent Jewel Canson of the PNP, and the
petitioner and intervenors as among the members.
Acting on a media expose of SPO2 Eduardo delos Reyes, a member of the CIC, that what
actually transpired at dawn of May 18, 1995 was a summary execution (or a rub out) and
not a shoot-out between the Kuratong Baleleng gang members and the ABRITG.
Ombudsman Aniano Desierto investigated the case which led to the filing of eleven (11)
informations for multiple murder with the Sandiganbayan.
On March 5-6, 1996, all the accused filed separate motions questioning the jurisdiction of
the Sandiganbayan, asserting that under the amended informations, the cases fall within
the jurisdiction of the Regional Trial Court.
The Sandiganbayan admitted the amended information and ordered the cases
transferred to the Quezon City Regional Trial Court which the Office of the Special
Prosecutor moved for a reconsideration, insisting that the cases should remain with the
Sandiganbayan.
Subsequently, the Sandiganbayan promulgated a Resolution denying the motion for
reconsideration of the Special Prosecutor, ruling that it "stands pat in its resolution…”
On the same day, however, the Sandiganbayan issued and ADDENDUM to its Resolution,
stating that the said Court has competence to take cognizance of these cases. The
multiple murder charge against petitioner and intervenors falls under Section 4
[paragraph b] of R.A. 8249. Section 4 requires that the offense charged must be
committed by the offender in relation to his office in order for the Sandiganbayan to have
jurisdiction over it. This jurisdictional requirement is in accordance with Section 5,
Article XIII of the 1973 Constitution which mandated that the Sandiganbayan shall have
jurisdiction over criminal cases committed by the public officers and employees,
including those in goverment-owned or controlled corporations, "in relation to their
office as may be determined by law."
The amended informations however, appears to be wanting of specific factual averments
to show the intimate relation/connection between the offense charged and the discharge
of official function of the offenders.
ISSUE:

Did the amended informations successfully establish the intimate connection


between the offense charged and the discharge of official functions of the PNP officers,
which is an essential factor to determine whether the case falls within the
Sandiganbayan’s or Regional Trial Court’s jurisdiction?

RULING:
No, the failure to establish in the amended informations that the charge of murder
was intimately connected with the discharge of official functions of the accused PNP
officers, the offense charged in the subject criminal cases is plain murder and, therefore,
within the exclusive original jurisdiction of the Regional Trial Court, not the
Sandiganbayan.
In People vs. Montejo, it was held that an offense is said to have been committed in
relation to the office if it (the offense) is "intimately connected" with the office of the
offender and perpetrated while he was in the performance of his official functions. This
intimate relation between the offense charged and the discharge of official duties "must
be alleged in the informations."
As to how the offense charged be stated in the informations, Section 9, Rule 110 of the
Revised Rules of Court mandates:
Sec. 9 Couse of accusation — The acts or omissions complied of as constituting the
offense must be stated in ordinary and concise language without repetition not
necessarily in the terms of the statute defining the offense, but in such from as is
sufficient to enable a person of common understanding to know what offense is intended
to be charged, and enable the court to pronounce proper judgment. (Emphasis supplied)
As early as 1954, the Supreme Court pronounced that "the factor that characterizes the
charge is the actual recital of the facts." The real nature of the criminal charge is
determined not from the caption or preamble of the informations nor from the
specification of the provision of law alleged to have been violated, they being
conclusions of law, but by the actual recital of facts in the complaint or information.
Applying these, the Court finds the amended informations for murder against herein
petitioner and intervenors wanting of specific factual averments to show the intimate
relation/connection between the offense charged and the discharge of official function of
the accused PNP officers. The mere allegation in the amended information that the
offense was committed by the accused public officer in relation to his office is not
sufficient.

JOSE T. OBOSA vs. CA

FACTS: Senior State Prosecutor Trampe charged the accused Jose Obosa and three others
with murder on two counts with the Regional Trial Court of Makati, for the ambush-slaying of
Secretary of Local Governments Jaime N. Ferrer and his driver Jesus D. Calderon. At that time,
Ferrer was riding in his car, going to the St. Andrew Church near the plaza of La Huerta, to hear
Sunday mass. Each information alleged that the killing was with the attendance of the following
qualifying/aggravating circumstances, to wit: treachery, evident premeditation, abuse of superior
strength, nighttime purposely sought, disregard of the respect due to the victim on account of
his rank and age (as to Secretary Ferrer), and by a band. The Prosecutor recommended no
bail, as the evidence of guilt was strong. During the trial of the two cases, which were
consolidated and tried jointly, the accused Obosa was detained at Camp Bagong Diwa, Taguig,
Metro Manila. At the time of the commission of the two offenses, the accused Obosa was a
virtual "escapee" from the National Penitentiary at Muntinlupa, Metro Manila. Indeed, by virtue
of a subpoena illegally issued by a judge of the MTC of Quezon, accused Obosa was escorted
out of prison to appear before said judge on the pretext that the judge needed his presence.
While accused Obosa was out of prison, he was able to participate in the commission of the
double murder now charged against him as principal for the ambush-slaying of Secretary Ferrer
and his driver. Witnesses positively identified accused Obosa as one of three assassins. RTC
found the accused Obosa guilty beyond reasonable doubt of homicide on two counts. In ruling
that the crime committed was homicide, not murder as charged in the informations, the lower
court declared that there was no qualifying circumstance attendant. In fact, however, the
lower court itself found that the accused shot the victims while the latter were inside the car,
unwary of any danger to their lives, for unknown to them, were the assassins lurking in the dark,
firing their guns from behind, a circumstance showing treachery. There is treachery when the
victims were attacked without warning and their backs turned to the assailants. There is
treachery when the unarmed and unsuspecting victim was ambushed in the dark, without any
risk to his assailants. Moreover, the crimes could be qualified by taking advantage of superior
strength and aid of armed men. Where the attackers cooperated in such a way to secure
advantage of their combined strength, there is present the qualifying circumstance of taking
advantage of superior strength. The lower court promulgated its decision and on the same
occasion, accused Obosa manifested his intention to appeal and asked the Court to allow him
to post bail for his provisional liberty. Immediately, the lower court granted accused Obosa's
motion and fixed bail at P20,000.00, in each case. Accused Obosa filed a written notice of
appeal, thereby perfecting appeal from the decision. By the perfection of the appeal, the lower
court thereby lost jurisdiction over the case and this means both the record and the
person of the accused-appellant. On June 4, 1990, accused Obosa filed a bailbond in the
amount of P40,000.00, through Plaridel Surety and Assurance Company, which the lower court
approved. On the same day, June 4, 1990, the lower court issued an order of release. The
prison authorities at the National Penitentiary released accused Obosa also on the same day
notwithstanding that, at the time of the commission of the double murder, accused Obosa was
serving a prison term for robbery. The respondent Court likewise discoursed on the service of
sentence made by the accused. Thus, it extensively discussed the following computation on the
penalties imposed upon the petitioner for his previous offenses, which all the more convinced
respondent Court that petitioner was not entitled to bail on the date he applied therefor on May
31, 1990 and filed his bailbond on June 4, 1990, as follows: 12At the time the accused
committed the crimes charged, he was an inmate at the National Penitentiary, New Bilibid
Prisons, Muntinlupa, Metro Manila. He was in jail, but was able to commit the Ferrer
assassination. He was serving imprisonment by final judgment in each of three such as theft,
robbery and evasion of sentence. These sentences are to be served successively not
simultaneously. He also forfeited his allowance for good conduct prescribed by law.Respondent
People, through the Office of the Solicitor General (OSG), filed with respondent Court an urgent
motion, 13 praying for cancellation of petitioner's bail bond.

ISSUE: Whether or not petitioner is entitled to bail pending appeal from judgment convicting him
of homicide though charged with murder.

RULING: SC ruled that bail cannot be granted as a matter of right even after an accused, who
is charged with a capital offense, appeals his conviction for a non-capital crime. Courts must
exercise utmost caution in deciding applications for bail considering that the accused on appeal
may still be convicted of the original capital offense charged and that thus the risk attendant to
jumping bail still subsists. In fact, trial courts would be well advised to leave the matter of bail,
after conviction for a lesser crime than the capital offense originally charged, to the appellate
court's sound discretion. SC hold that the trial court had failed to exercise the degree of
discretion and caution required under and mandated by our statutes and rules, for, aside from
being too hasty in granting bail immediately after promulgation of judgment, and acting without
jurisdiction in approving the bailbond, it inexplicably ignored the undeniable fact of petitioner's
previous escape from legal confinement as well as his prior convictions. Upon the other hand,
the respondent Court should be commended for its vigilance, discretion and steadfastness. In
ruling against bail, it even scoured the records and found that treachery attended the killing
thereby justifying its action. The trial court's literal interpretation of the law on bail was forcefully
debunked by the appellate courts' excellent disquisition on the rationale of the applicable rules.
Truly, law must be understood not by "the letter that killeth but by the spirit that giveth life." Law
should not be read and interpreted in isolated academic abstraction nor even for the sake of
logical symmetry but always in context of pulsating social realities and specific environmental
facts. Truly, "the real essence of justice does not emanate from quibblings over patchwork legal
technicality. It proceeds from the spirit's gut consciousness of the dynamic role of law as a brick
in the ultimate development of the social edifice."

Executive Summary

Question: Can a peitioner who is convicted of homicide under RTC, although charged with
murder, pending appeal, be entitled of bail as a matter of right?

Answer: No. Bail cannot be granted as a matter of right even after an accused, who is charged
with a capital offense, appeal his conviction for a non-capital crime. Courts must exercise
utmost caution in deciding applications for bail considering that the accused on appeal may still
be convicted of the capital offense charged and thus, the risk attendant to jumping bail still
subsists. In fact, trial courts are advised to leave the decision on the said situation to the
appellate court's sound discretion

G.R. No. 179817


ANTONIO F. TRILLANES IV vs. HON. OSCAR PIMENTEL, SR., IN HIS CAPACITY AS
PRESIDING JUDGE, REGIONAL TRIAL COURT- BRANCH 148, MAKATI CITY; GEN.
HERMOGENES ESPERON, VICE ADM. ROGELIO I. CALUNSAG, MGEN. BENJAMIN
DOLORFINO, AND LT. COL. LUCIARDO OBEÑA
June 27, 2008

DOCTRINE: Emergency or compelling temporary leaves from imprisonment are allowed to all


prisoners, at the discretion of the authorities or upon court orders.

FACTS: At the wee hours of July 27, 2003, a group of more than 300 heavily armed soldiers led
by junior officers of the Armed Forces of the Philippines (AFP) stormed into the Oakwood
Premier Apartments in Makati City and publicly demanded the resignation of the President and
key national officials.
Later in the day, President Gloria Macapagal Arroyo issued Proclamation No. 427 and General
Order No. 4 declaring a state of rebellion and calling out the Armed Forces to suppress the
rebellion. A series of negotiations quelled the teeming tension and eventually resolved the
impasse with the surrender of the militant soldiers that evening.

In the aftermath of this eventful episode dubbed as the "Oakwood Incident," petitioner Antonio
F. Trillanes IV was charged, along with his comrades, with coup d’etat defined under Article
134-A of the Revised Penal Code before the Regional Trial Court (RTC) of Makati.

Close to four years later, petitioner, who has remained in detention, threw his hat in the political
arena and won a seat in the Senate with a six-year term commencing at noon on June 30,
2007.

Before the commencement of his term or on June 22, 2007, petitioner filed with the RTC, Makati
City, Branch 148, an "Omnibus Motion for Leave of Court to be Allowed to Attend Senate
Sessions and Related Requests" 

ISSUE: Whether or not there are enough precedents to allow liberal treatment of detention
prisoners who are held without bail as in the case of former President Estrada and former
ARMM Gov. Nur Misuari?

RULING: No.

In arguing against maintaining double standards in the treatment of detention prisoners,


petitioner expressly admits that he intentionally did not seek preferential treatment in the form of
being placed under Senate custody or house arrest, yet he at the same time, gripes about the
granting of house arrest to others.

Emergency or compelling temporary leaves from imprisonment are allowed to all prisoners, at


the discretion of the authorities or upon court orders. That this discretion was gravely abused,
petitioner failed to establish. In fact, the trial court previously allowed petitioner to register as a
voter in December 2006, file his certificate of candidacy in February 2007, cast his vote on May
14, 2007, be proclaimed as senator-elect, and take his oath of office on June 29, 2007. In a
seeming attempt to bind or twist the hands of the trial court lest it be accused of taking a
complete turn-around, petitioner largely banks on these prior grants to him and insists on
unending concessions and blanket authorizations.
Petitioner’s position fails. On the generality and permanence of his requests alone, petitioner’s
case fails to compare with the species of allowable leaves. Jaloslos succinctly expounds:
x x x Allowing accused-appellant to attend congressional sessions and committee
meetings for five (5) days or more in a week will virtually make him a free man with all
the privileges appurtenant to his position. Such an aberrant situation not only elevates
accused-appellant’s status to that of a special class, it also would be a mockery of the
purposes of the correction system.

DEONINO UY REM REV II CASE DIGEST


No. 24 BARBERO VS. SANTIAGO
Doctrine: The Accused must be discharged upon approval of the bail by the judge with whom it
was filed and where the case is pending.

Facts: The Petitioner filed a criminal case for estafa against a certain Herman Medina. The
case was raffled to the RTC branch 36 in Santiago, Isabela. The Presiding Judge issued a
warrant of arrest against Medina, and by virtue of the Warrant of Arrest Medina was arrested.

After his arrest Median filed a petition for bail, which was approved by the respondent Judge
whop is the Presiding Judge of a Municipal Trial Court in San Mateo, Isabela, and commanded
the BJMP and the PNP to release Medina.

Petitioner questioned the approval of bail by the respondent Judge for being unlawful. As a
result the Petitioner filed a complaint for ignorance of the law against the respondent Judge
before the Office of the Court Administrator (OCA). The OCA directed Judge Dumlao to comment
on the complaint, but he ignored and did not file his comment.

The OCA finds Judge Dumlao liable for gross ignorance of the law and for violation of Court
directive.

ISSUE: WON RESPONDENT JUDGE IS GUILTY OF SERIOUS AND GRAVE ABUSE OF


DISCRETION?

RULING: Yes, the Respondent Judge Dumlao is guilty of grave abuse of discretion.

Section 3, Rule 114 of the Rules of Court provides that no person under detention by legal
process shall be released except when he is admitted to bail. Section 19 provides that the
accused must be discharged upon approval of the bail by the judge with whom it was filed in
accordance with Section 17. Section 17 provides that the bail may be filed with the court
where the case is pending, unless (1) the judge in that court is absent or unavailable, or (2)
the accused is arrested in a province, city, or municipality other than where the case is pending.
If the judge is absent or unavailable, the bail should be filed with another branch of the same
court. If the accused is arrested in a province, city, or municipality other than where the case is
pending, the bail should be filed with any RTC of the place.

In the case at bar, there was no showing that Judge Anghad was absent or unavailable or that
Medina was arrested outside Santiago City. Thus, Medina's bail should have been filed with
Judge Anghad. Even if Judge Anghad were absent or unavailable or even if Medina were
arrested in San Mateo, Judge Dumlao would still be liable because the bail should have been
filed with another branch of the RTC in Santiago City or with the RTC of San Mateo,
respectively

Since the criminal case was pending before the RTC of Santiago City and there was no showing
that Judge Anghad of the RTC was absent or unavailable, Judge Dumlao lacked authority to
approve the bail and order Medina's release.

JUDGE DIVINA LUZ P. AQUINO-SIMBULAN vs. PRESIDING JUDGE NICASIO


BARTOLOME (retired), ACTING CLERK OF COURT ROMANA C. PASCUAL, CLERK OF
COURT MILAGROS P. LEREY (retired), and DOCKET CLERK AMOR DELA CRUZ, all of
the Municipal Trial Court, Sta. Maria, Bulacan
DOCTRINE: Where the accused was not arrested, the proper procedure, according to the rules,
would have been to file her bail bond with the RTC Branch 41, San Fernando, Pampanga
where her case was pending. Had complainant Judge been absent or was unavailable at that
time, the accused could file for bail with another branch of the RTC in Pampanga or in San
Fernando City.

FACTS: Criminal Case No. 13360 was originally raffled to the Regional Trial Court (RTC),
Branch 41, San Fernando, Pampanga, where complainant Judge presides whose branch
received an Indorsement from Warrant/Subpoena Officer PO3 Edwin Villacentino of the
Sasmuan Municipal Police Station stating that the accused Mercado voluntarily surrendered
before the MTC of Sta. Maria, Bulacan and posted her bail bond through Summit Guaranty &
Insurance Co., Inc., which was duly approved by respondent Judge. This prompted complainant
to issue an Order directing respondent Lerey, then Clerk of Court of the MTC, to transmit to the
RTC within twenty-four (24) hours from receipt of said Order, the bond which the former court
approved. However, the Clerk of Court failed to comply, complainant Judge issued an Order
dated January 12, 2004 directing the former to explain in writing within three (3) days from
receipt thereof why she should not be cited in contempt for delaying the administration of
justice. RTC received a letter from respondent Romana Pascual, then Acting Clerk of Court of
the MTC, explaining that the bail bond in Criminal Case No. 13360 was approved by respondent
Judge during the tenure of Lerey, and that the latter had already retired. Thereafter, the RTC
received a written explanation from Lerey stating that she had misplaced and overlooked the
subject surety bond, which resulted in the delay of its transmission to the RTC. Upon perusal of
the documents Lerey’s documents, complainant Judge discovered that the subject surety bond
bore some erasures, and its attachments were highly anomalous. In view of these findings, the
RTC issued a subpoena to respondents Pascual and Lerey directing them to appear before it to
explain the aforementioned errors.
During the hearing, respondents Pascual and Lerey appeared before the RTC, Branch 41, San
Fernando, Pampanga, and the following facts were established therein:
1. That respondent Judge issued an Order of Release dated August 21, 2003 without a
Certificate of Detention and Warrant of Arrest attached to the documents presented to him;
2. That while the Order of Release was dated August 21, 2003, the Undertaking and
Certification from the bonding company were dated November 22, 2003 and October 29, 2003,
respectively;
3. That it was Lerey who reviewed the documents before the surety bond was referred to
respondent Judge for the latter’s approval; and
4. That the delay in the transmission of the bond and its supporting documents was attributed to
Amor dela Cruz, Docket Clerk of the MTC of Sta. Maria, Bulacan.

After the hearing, Public Prosecutor Otto Macabulos stated that he found the explanation too
shallow and self-serving, and that he would file an indirect contempt case under Rule 71,
Section 3 (d) of the 1997 Rules of Civil Procedure against Lerey and Dela Cruz. The Court
referred the Report of the Investigating Judge to the OCA for evaluation, report and
recommendation within thirty (30) days from receipt of records.

DCA Jose P. Perez observed that:


1. In approving the surety bond of the accused, respondent Judge violated Section 17, Rule 114
of the Rules of Court In. the instant case, the accused Rosalina Mercado was not arrested. That
being the case, she should have filed her bail bond with the court where her case was pending,
i.e., the Regional Trial Court, Branch 41, San Fernando City, Pampanga. In the absence of the
judge thereof, it could be done at another branch of the same court within the province of
Pampanga or City of San Fernando. Instead, accused Mercado filed her bond in the Municipal
Trial Court of Sta. Maria, Bulacan, where respondent Judge presides, who approved the same
and ordered her release from custody.

2. Respondent Judge did not require the accused to submit the supporting documents pertinent
to the application for a bond. It appears that there was no Certificate of Detention presented to
him; hence, there was no legal justification for him to issue the Order of Release and process
the bond since the accused was not detained within his jurisdiction. Also, there was no Warrant
of Arrest attached to the documents presented to him. Moreover, all the supporting papers were
belatedly filed: (a) Undertaking was dated 22 November 2003; (b) Certification from the Office of
the Court Administrator was dated 29 October 2003; and (c) the Certification from Summit
Guaranty & Insurance Co., Inc. was dated 22 November 2003.

ISSUE: Whether or not there were indeed grave errors and discrepancies committed by
respondents Judge Bartolome and Lerey in processing the surety bond for the accused in
Criminal Case No. 13360

RULING: Yes. After a careful evaluation of the records and the Reports of the Investigating
Judge and the OCA, the Court holds that there were indeed grave errors and discrepancies
committed by respondents Judge Bartolome and Lerey in processing the surety bond for the
accused in Criminal Case No. 13360.

The following provisions of the Revised Rules of Criminal Procedure apply before an accused
can be released on bail:
Sec. 14. Bail, where filed. (a) Bail in the amount fixed may be filed with the court where the case
is pending, or, in the absence or unavailability of the judge thereof, with another branch of the
same court within the province or city. If the accused is arrested in a province, city or
municipality other than where the case is pending, bail may be filed also with any regional trial
court of said place, or, if no judge thereof is available, with any metropolitan trial judge,
municipal trial judge or municipal circuit trial judge therein. x x x

Sec. 16. Release on bail. The accused must be discharged upon approval of the bail by the
judge with whom it was filed in accordance with Section 14 hereof.

Whenever bail is filed with a court other than where the case is pending, the judge accepting the
bail shall forward the bail, the order of release and other supporting papers to the court where
the case is pending, which may, for good reason, require a different one to be filed.

The OCA’s Report revealed that the accused Rosalina Mercado was not arrested. The proper
procedure, according to the above-cited rules, would have been to file her bail bond with the
RTC Branch 41, San Fernando, Pampanga where her case was pending. Had complainant
Judge been absent or was unavailable at that time, the accused could file for bail with another
branch of the RTC in Pampanga or in San Fernando City. However, the accused filed her surety
bond with the MTC of Sta. Maria, Bulacan, where it was approved by respondent Judge.
Not only did respondent Judge erroneously order the release of the accused, but he also failed
to require submission of the supporting documents needed in the application for a bond. There
was no Certificate of Detention or Warrant of Arrest attached to the bond transmitted by the
MTC to the complainant Judge. Moreover, the other supporting documents were belatedly filed.
Records show that respondent Judge approved the bail bond on August 21, 2003, but the
Undertaking was dated November 22, 2003, the Certification from the OCA was dated October
29, 2003, and the Certification from Summit Guaranty and Insurance Co., Inc. was dated
November 22, 2003.
REGINO SY CATIIS vs. COURT OF APPEALS, REYNALDO A. PATACSIL, ENRICO D.
LOPEZ, LUZVIMINDA A. PORTUGUEZ and THE BUREAU OF JAIL MANAGEMENT AND
PENOLOGY, NATIONAL CAPITAL REGION, MAKATI CITY JAIL, THROUGH ITS OFFICER-
IN-CHARGE WARDEN, CHIEF INSP. ISAGANI M. GAMINO
G.R. NO. 153979. February 6, 2006.

DOCTRINE:

All persons, except those charged with offenses punishable by reclusion perpetua when
evidence of guilt is strong, shall before conviction, be bailable by sufficient sureties or be
released on recognizance as may be provided by law.

FACTS:

Petitioner filed a letter-complaint against private respondents Reynaldo A. Patacsil, Enrico D.


Lopez, Luzviminda A. Portuguez and Margielyn Tafalla for violation of Art. 315 of the Revised
Penal Code in relation to Presidential Decree No. 1689 (syndicated estafa) and other related
offenses. A resolution was then issued by the assistant city prosecutor finding the existence of a
probable cause for syndicated estafa against private respondents with no bail recommended.
An Information was also filed by the city prosecutor against private respondents which stated
that the aforementioned accused committed estafa in a syndicated manner consisting of five or
more persons.

Thereafter, Judge Bersamin issued an Order finding probable cause against all the accused and
approved the recommendation of the City Prosecutor that the charge be non-bailable. When a
notice of hearing was then issued setting the case for arraignment, private respondents filed an
urgent motion to fix bail. Upon arraignment, they also entered pleas of not guilty.

Judge Bersamin then issued another order recanting his approval of the recommendation to
deny bail. According to him, the information only charged four persons. In order to qualify as
syndicated estafa which has a penalty of life imprisonment to death, the offense must have
been committed by five or more persons. Further, Judge Bersamin also noted that since no
aggravating circumstance had been alleged in the information, it precludes the State from
proving any aggravating circumstance which will raise the penalty to its maximum period
of reclusion perpetua.

The Court of Appeals found that the assailed order of Judge Bersamin cannot be characterized
as one issued with grave abuse of discretion for he correctly determined that the Information did
not charge a syndicated estafa; that with only four charged in the information, it could not be
considered as committed by a syndicate which must consist of five or more persons and he
cannot be faulted for that.

ISSUES:

1. Whether Judge Bersamin is correct in finding that the Information did not charge a
syndicated estafa
2. Whether Judge Bersamin is correct in finding that the crime charged is bailable despite
that the imposable penalty ranges from reclusion temporal to reclusion perpetua

HELD:

1. Yes. P.D. No. 1689 defined a syndicate as “consisting of five or more persons formed
with the intention of carrying out the unlawful or illegal act, transaction, enterprise or
scheme”, and such definition is controlling. Where a requirement is made in explicit and
unambiguous terms, no discretion is left to the judiciary.

In this case, the Information specifically charged only four persons without specifying
any other person who had participated in the commission of the crime charged, thus,
based on the definition of syndicate under the law, the crime charged was not committed
by a syndicate. The wordings in the information that the accused conspired with each
other "in a syndicated manner consisting of five (5) or more persons…” is not sufficient
compliance with the requirements of the law on what constitute a syndicate.

2. Yes. It is a requirement that the aggravating as well as the qualifying circumstances be


expressly and specifically alleged in the complaint or information. Otherwise, they cannot
be considered by the trial court in their judgment, even, if they are subsequently proved
during trial. 

A reading of the Information, however, shows that there was no allegation of any
aggravating circumstance, thus Judge Bersamin is correct when he found that the lesser
penalty, i.e., reclusion temporal, is imposable in case of conviction.

Section 13, Article III of the Constitution provides that all persons, except those charged
with offenses punishable by reclusion perpetua when evidence of guilt is strong, shall
before conviction, be bailable by sufficient sureties or be released on recognizance as
may be provided by law. In pursuance thereof, Section 4 of Rule 114, as amended, now
provides that all persons in custody shall, before conviction by a regional trial court of an
offense not punishable by death, reclusion perpetua or life imprisonment, be admitted to
bail as a matter of right. Since the imposable penalty on private respondents, in case of
conviction, is reclusion temporal, they are entitled to bail as a matter of right.

PEOPLE OF THE PHILIPPINES, Petitioner, vs. LUIS PLAZA Y BUCALON, Respondent.

G.R. No. 176933, October 2, 2009

Doctrine:

Section 13, Article III of the Constitution provides that "All persons, except those charged with
offenses punishable by reclusion perpetua when evidence of guilt is strong, shall, before
conviction, be bailable by sufficient sureties, or be released on recognizance as may be
provided by law."
Section 4 of Rule 114 of the Revised Rules of Court, as amended, thus provides that all persons
in custody shall, before conviction by a regional trial court of an offense not punishable by
death, reclusion perpetua or life imprisonment, be admitted to bail as a matter of right.

The exercise by the trial court of its discretionary power to grant bail to an accused charged with
a capital offense thus depends on whether the evidence of guilt is strong.

Facts:

The case was originally raffled to Branch 30 of the Surigao RTC presided by Judge Floripinas
Buyser (Judge Buyser). After the prosecution rested its case, respondent, with leave of court,
filed a Demurrer to Evidence. The Demurrer was denied by Judge Buyser by Order of March 14,
2002.

The defense thereupon presented evidence in the course of which respondent filed a Motion to
Fix Amount of Bail Bond, contending that in view of Judge Buyser’s ruling that the prosecution
evidence is sufficient to prove only Homicide, he could be released on bail. He thus prayed that
the bail bond for his temporary liberty be fixed at ₱40,000.00 which he claimed was the usual
bond for Homicide in the RTC of Surigao City and Surigao del Norte.

In its Opposition to Motion to Fix Amount of Bail Bond, the prosecution contended, in the main,
that the case being for Murder, it is non-bailable as the imposable penalty is reclusion temporal
to death; that it is the public prosecutor who has exclusive jurisdiction to determine what crime
the accused should be charged with; that the accused should have filed a motion/application to
bail and not just a motion to fix the amount of the bail bond; that the accused had already
waived his right to apply for bail at that stage of the proceedings; that Judge Buyser’s March 14,
2002 Order, being a mere opinion and not a ruling or a dispositive part thereof, produced no
legal effect inasmuch as it had no jurisdiction to rule on a matter outside the Demurrer; and that
under the Rules, the prosecution could still prove the existence of treachery on rebuttal after the
defense has rested its case.

During the hearing of the Motion to Fix Amount of Bail Bond, Senior State Prosecutor Rogelio
Bagabuyo questioned Judge Buyser’s impartiality, prompting the judge to inhibit himself and to
order the case transferred to Branch 29 of the RTC for further proceedings.
Branch 29 Presiding Judge Jose Manuel Tan (Judge Tan) heard the Motion to Fix Amount of
Bail Bond.

By Order 7 of November 12, 2002, Judge Tan, concurring with the finding of Judge Buyser that
since the prosecution evidence proved only Homicide which is punishable by reclusion temporal
and, therefore, bailable, ruled that respondent could no longer be denied bail. He accordingly
granted respondent’s Motion and fixed the amount of his bond at ₱40,000.

Petitioner’s motion for reconsideration cum prayer for inhibition of Judge Tan was denied for
lack of merit. Respondent was subsequently released after he posted a ₱40,000 bond.

Roberto Murcia (Roberto), the victim’s brother, impleading the People as co-petitioner, assailed
the trial court’s orders via petition for certiorari with the Court of Appeals.

Roberto faulted Judge Tan for granting bail without an application for bail having been filed by
respondent and without conducting the mandatory hearing to determine whether or not the
prosecution’s evidence is strong.

By Decision of January 31, 2007, the appellate court, observing that the allegations in
respondent’s Motion to Fix Amount of Bail Bond constituted an application for bail, dismissed
Roberto’s petition and affirmed Judge Tan’s orders. Hence this petition.

Issue:

Whether or not the respondent Luis B. Plaza is entitled to bail.

Ruling:

Yes. Section 13, Article III of the Constitution provides that "All persons, except those charged
with offenses punishable by reclusion perpetua when evidence of guilt is strong, shall, before
conviction, be bailable by sufficient sureties, or be released on recognizance as may be
provided by law."
Section 4 of Rule 114 of the Revised Rules of Court, as amended, thus provides that all persons
in custody shall, before conviction by a regional trial court of an offense not punishable by
death, reclusion perpetua or life imprisonment, be admitted to bail as a matter of right.

The exercise by the trial court of its discretionary power to grant bail to an accused charged with
a capital offense thus depends on whether the evidence of guilt is strong. Stressing this point,
this Court held:

. . . [W]hen bail is discretionary, a hearing, whether summary or otherwise in the discretion of


the court, should first be conducted to determine the existence of strong evidence or lack of it,
against the accused to enable the judge to make an intelligent assessment of the evidence
presented by the parties. A summary hearing is defined as "such brief and speedy method of
receiving and considering the evidence of guilt as is practicable and consistent with the purpose
of hearing which is merely to determine the weight of evidence for the purposes of bail." On
such hearing, the court does not sit to try the merits or to enter into any nice inquiry as to the
weight that ought to be allowed to the evidence for or against the accused, nor will it speculate
on the outcome of the trial or on what further evidence may be therein offered and admitted.
The course of inquiry may be left to the discretion of the court which may confine itself to
receiving such evidence as has reference to substantial matters, avoiding unnecessary
examination and cross examination." 13 (Emphasis and underscoring supplied)

Since Judge Tan concurred with the assessment by Judge Buyser of the prosecution evidence
when he denied the Demurrer and the latter’s statement that the evidence was sufficient to
convict respondent of Homicide, holding a summary hearing merely to determine whether
respondent was entitled to bail would have been unnecessary as the evidence in chief was
already presented by the prosecution.
GAUDENCIO B. PANTILO III, complainant, vs. JUDGE VICTOR A. CANOY, respondent.
February 9, 2011

DOCTRINE

Sec. 17, Rule 114 of the Revised Rules on Criminal Procedure allows that any person in
custody who is not yet charged in court “may apply for bail with any court in the province, city or
municipality where he is held.” In the case at bar, Melgazo did not file any application or petition
for the grant of bail with the Surigao City RTC, Branch 29. Despite the absence of any written
application, respondent judge verbally granted bail to Melgazo. This is a clear deviation from the
procedure laid down in Sec. 17 of Rule 114.

FACTS

The complainant, Pantilo, the brother of the homicide victim recounts in his letter-complaint that,
on September 3, 2008, at around 5 o’clock in the afternoon, he, along with police officers acting
as escorts of Leonardo Luzon Melgazo (Melgazo), the accused, went to the City Prosecutor’s
Office, Surigao City, to attend the inquest proceedings. Later, at around 8 o’clock in the evening,
Pantilo was informed that Melgazo had been released from detention.

The following day, Pantilo went to the Police Station to verify the information. Upon arriving
there, A Custodial Officer told him that Melgazo had indeed been released as shown in the
Police Logbook and as authorized by Chief of Police .Further, the logbook showed that Melgazo
was temporarily released upon the order of Judge Canoy after he posted bail in the amount of
thirty thousand pesos (PhP 30,000).

Pantilo proceeded to the Office of the Clerk of Court to request a copy of the Information, only to
find out that none had yet been filed by the Surigao City Prosecutor’s Office, he inquired from
the City Prosecutor’s Office the details surrounding the release of Melgazo. He learned that no
Information had yet been filed in Court that would serve as the basis for the approval of the bail.
Likewise, he also learned from the City Police Station that no written Order of Release had been
issued but only a verbal order directing the police officers to release Melgazo from his detention
cell. Melgazo filed a Motion for the Release of his impounded vehicle as physical evidence
pending the trial of the case which was granted by the Judge.

Subsequently, Pantilo filed a motion for inhibition of Judge Canoy which was later denied.
Aggrieved, Pantilo filed a letter-complaint before the Office of the Court Administrator charging
Judge Canoy with (1) gross ignorance of the law and procedures; (2) grave abuse of authority;
and (3) appearance of impropriety (Canon 2, Code of Judicial Conduct). Pantilo also prays for
Judge Canoy’s disbarment.

ISSUE

May a judge verbally grant bail (“constructive bail”) to the accused who is not yet charged and
did not file an application or petition for its grant? NO

RULING
It is settled that an accused in a criminal case has the constitutional right to bail, more so in this
case when the charge against Melgazo, Reckless Imprudence Resulting in Homicide, is a non-
capital offense. However, the letter-complaint focuses on the manner of Melgazo’s release from
detention.

Sec. 17, Rule 114 of the Revised Rules on Criminal Procedure allows that any person in
custody who is not yet charged in court "may apply for bail with any court in the province, city or
municipality where he is held." In the case at bar, Melgazo did not file any application or petition
for the grant of bail. Despite the absence of any written application, respondent judge verbally
granted bail to Melgazo. This is a clear deviation from the procedure laid down in Sec. 17 of
Rule 114.

Also there was also a violation of Sec. 14 of Rule 114, Melgazo or any person acting in his
behalf did not deposit the amount of bail recommended by Prosecutor Gonzaga with the nearest
collector of internal revenue or provincial, city or municipal treasurer. In clear departure from
Sec. 14 of Rule 114, Judge Canoy instead verbally ordered Clerk IV Suriaga of the Surigao City
RTC, Office of the Clerk of Court, to accept the cash deposit as bail, to earmark an official
receipt for the cash deposit, and to date it the following day. Worse, respondent judge did not
require Melgazo to sign a written undertaking containing the conditions of the bail under Sec. 2,
Rule 114 to be complied with by Melgazo. Immediately upon receipt by Suriaga of the cash
deposit of PhP 30,000 from Melgazo, Judge Canoy ordered the police escorts to release
Melgazo without any written order of release. In sum, there was no written application for bail,
no certificate of deposit from the BIR collector or provincial, city or municipal treasurer, no
written undertaking signed by Melgazo, and no written release order.

As regards the insistence of Judge Canoy that such may be considered as "constructive bail,"
there is no such species of bail under the Rules. Despite the noblest of reasons, the
Rules of Court may not be ignored at will and at random to the prejudice of the rights of
another.

Procedural rules have their own wholesome rationale in the orderly administration of justice.
Justice has to be administered according to the Rules in order to obviate arbitrariness, caprice,
or whimsicality." In other words, "rules of procedure are intended to ensure the orderly
administration of justice and the protection of substantive rights in judicial and extrajudicial
proceedings." In this case, the reason of Judge Canoy is hardly persuasive enough to disregard
the Rules.

From the foregoing, the Court finds Judge Canoy guilty of a less serious charge of violation of
Supreme Court rules, directives and circulars under Sec. 9, Rule 140. He is STERNLY
WARNED that a repetition of similar or analogous infractions in the future shall be dealt with
more severely.
PANFILO D. BONGCAC, petitioner, vs. SANDIGANBAYAN, PEOPLE OF THE
PHILIPPINES, SPECIAL PROSECUTOR FORTUNATO LIM, and TORIBIO BON,
respondents.

Doctrine:  Bail; The cancellation of bail is automatic upon execution of the judgment of
conviction.

Facts: Panfilo D. Bongcac was designated by the Mayor of Tagbilaran City, Jose Torralba as
the mayor’s representative to the city market committee ,consultant and coordinator and adviser
to the acting market administrator.
Eventually, Egineer Fortunato Lim and Toribio Bon applied for the stalls as tiendas in the Cogon
Public Market in Tagbilaran City.
Further, Panfilo included Lim and Toribio as awardees. Unfortunately, the latter were informed
that the city government could not afford to construct a new market and if they interested they
should give him more money for the construction of the stalls.
Accordingly, Lim and Toribio issued a check amounting to Php 92,000 pay to cash and
subsequently encashed by Panfilo. Consequently, the former was informed in a local
newspaper that petitoner was terminated as secretary to the mayor. They immediately,
demanded for the return of their money , but the petitioner failed to do so. Thus, he was charged
with two counts of Estafa before the Sandigabayan.
The Sandiganbayan found him guilty of estafa. He filed a motion for reconsideration however,
was denied. Thereafter, he filed a petition for review on certiorari and was also denied.

Finally, the Sandiganbayan issued a resolution and further directed the issuance of a bench
warrant of arrest against petitioner and to serve the sentenced imposed upon him. The cash
bond posted by the petitioner was order cancelled.

Issue: Whether the cancellation of bail by the Sandiganbayan proper.

Held: Yes, it is proper. Yes, On the cancellation of petitioner’s cash bail bond as ordered in the
Resolution of 10 January 2003 of the Sandiganbayan, the cancellation of the bail bond was due
to the execution of the final judgment of conviction. Section 22 of Rule 114 of the Revised Rules
of Criminal Procedure expressly provides:

“SEC. 22. Cancellation of bail.—Upon application of the bondsmen, with due notice to the


prosecutor, the bail may be cancelled upon surrender of the accused or proof of his death.
The bail shall be deemed automatically cancelled upon acquittal of the accused,
dismissal of the case, or execution of the judgment of conviction.
In all instances, the cancellation shall be without prejudice to any liability on the bail.”
(emphasis supplied).
From this provision, it is clear that the cancellation of bail is automatic upon execution of the
judgment of conviction. The Sandiganbayan did not err in cancelling petitioner’s cash bail bond
after the judgment of conviction became final and executory and its execution became
ministerial.
PANFILO D. BONGCAC, petitioner, vs. SANDIGANBAYAN, PEOPLE OF THE
PHILIPPINES, SPECIAL PROSECUTOR FORTUNATO LIM, and TORIBIO BON,
respondents.

NARCISO VS ROMANA-CRUZ, March 17 2010

DOCTRINE: Jurisprudence is replete with decisions compelling judges to conduct the required
hearings in bail applications, in which the accused stands charged with a capital offense. The
absence of objection from the prosecution is never a basis for the grant of bail in such cases, for
the judge has no right to presume that the prosecutor knows what he is doing on account of
familiarity with the case. “Said reasoning is tantamount to ceding to the prosecutor the duty of
exercising judicial discretion to determine whether the guilt of the accused is strong. Judicial
discretion is the domain of the judge before whom the petition for provisional liberty will be
decided. The mandated duty to exercise discretion has never been reposed upon the
prosecutor.”

FACTS:

After conducting a preliminary investigation on the death of Corazon Sta. Romana-Narciso, wife
of Joselito Narciso, Asst. City Prosecutor Myrna Dimaranan Vidal of Quezon City recommended
and thereafter filed the information for parricide against Joselito Narciso on November 13, 1991,
with the Regional Trial Court of Quezon City, docketed therein as Criminal Case No. Q-91-
24179.

Joselito Narciso thereafter asked for a review of the prosecutor’s resolution before the
Department of Justice (DOJ) which was however denied. Joselito Narciso moved for
reconsideration, which was still denied by the DOJ.

Failing before DOJ, the accused on February 6, 1992, filed in Criminal Case No. Q-91-24179 an
“Omnibus Motion for Reinvestigation and to Lift the Warrant of Arrest.” The Motion was granted
and the case was set for reinvestigation by another prosecutor.
Assistant Prosecutor Lydia A. Navarro, to whom the case was assigned for reinvestigation,
found no reason to disturb the findings of the previous prosecutor and recommended the
remand of the case to the court for arraignment and trial.

On August 3, 1992 the accused filed an Urgent Ex-Parte to Allow Accused Joselito Narciso to
Post Bail. The Public Prosecutor registered no objection and said motion was granted on the
same day, allowing the accused to post bail at P150,000.00.

The full text of the August 3, 1992 RTC Order, which the Court of Appeals annulled and set
aside, reads as follows:

“Accused who is present filed thru counsel a Motion to Allow Accused Joselito V. Narciso to
Post Bail. “Considering that the Presiding Judge of Branch 83 who is hearing this case is on
leave and the Pairing Judge Honorable Salvador Ceguerra is no longer within the premises,
there being no objection by the City Prosecutor Candido Rivera to the accused posting a
cashbond of P150,000.00, the undersigned in his capacity as Executive Judge hereby approves
the same.”

On August 14, 1992 the private prosecutor representing private complainant Flor Marie Sta.
Romana-Cruz, a sister of the accused's deceased wife, filed an Urgent Motion to Lift Order
Allowing Accused To Post Bail. Not obtaining any resolution on her Motion To Lift Order
Allowing Accused to Post Bail, private complainant filed Petition for Certiorari before the CA.

The Court of Appeals granted the petition.

The accused argued before the CA that he was entitled to bail because the evidence of his guilt
was not strong. He contended that the prosecutor’s conformity to his Motion for Bail was
tantamount to a finding that the prosecution evidence against him was not strong.

The Court of Appeals ruled, however, that there was no basis for such finding, since no hearing
had been conducted on the application for bail—summary or otherwise. The appellate court
found that only ten minutes had elapsed between the filing of the Motion by the accused and the
Order granting bail, a lapse of time that could not be deemed sufficient for the trial court to
receive and evaluate any evidence. 

ISSUE:
Whether or not the grant of bail is valid. 

HELD:

No, the grant of bail is not valid. SC agrees with CA.

Section 13, Article III of the Constitution, provides: 

“All persons, except those charged with offenses punishable by reclusion perpetua when
evidence of guilt is strong, shall, before conviction, be bailable by sufficient sureties, or be
released on recognizance as may be provided by law. The right to bail shall not be impaired
even when the privilege of the writ of habeas corpus is suspended. Excessive bail shall not be
required.” 

Furthermore, Section 7, Article 114 of the Rules of Court, as amended, also provides: 

“No person charged with a capital offense, or an offense punishable by reclusion perpetua or life
imprisonment, when evidence of guilt is strong, shall be admitted to bail regardless of the stage
of the criminal prosecution.”

Stressing in Basco v. Rapatalo that the judge had the duty to determine whether the evidence of
guilt was strong, the Court held:

“When the grant of bail is discretionary, the prosecution has the burden of showing that the
evidence of guilt against the accused is strong. However, the determination of whether or not
the evidence of guilt is strong, being a matter of judicial discretion, remains with the judge. ‘This
discretion by the very nature of things, may rightly be exercised only after the evidence is
submitted to the court at the hearing. Since the discretion is directed to the weight of the
evidence and since evidence cannot properly be weighed if not duly exhibited or produced
before the court, it is obvious that a proper exercise of judicial discretion requires that the
evidence of guilt be submitted to the court, the petitioner having the right of cross-examination
and to introduce his own evidence in rebuttal.’
“Consequently, in the application for bail of a person charged with a capital offense punishable
by death, reclusion perpetua or life imprisonment, a hearing, whether summary or otherwise in
the discretion of the court, must actually be conducted to determine whether or not the evidence
of guilt against the accused is strong. ‘A summary hearing means such brief and speedy
method of receiving and considering the evidence of guilt as is practicable and consistent with
the purpose of hearing which is merely to determine the weight of evidence for the purposes of
bail. On such hearing, the court does not sit to try the merits or to enter into any nice inquiry as
to the weight that ought to be allowed to the evidence for or against the accused, nor will it
speculate on the outcome of the trial or on what further evidence may be therein offered and
admitted. The course of inquiry may be left to the discretion of the court which may confine itself
to receiving such evidence as has reference to substantial matters, avoiding unnecessary
thoroughness in the examination and cross-examination.’ If a party is denied the opportunity to
be heard, there would be a violation of procedural due process.”

Jurisprudence is replete with decisions compelling judges to conduct the required hearings in
bail applications, in which the accused stands charged with a capital offense. The absence of
objection from the prosecution is never a basis for the grant of bail in such cases, for the judge
has no right to presume that the prosecutor knows what he is doing on account of familiarity
with the case. “Said reasoning is tantamount to ceding to the prosecutor the duty of exercising
judicial discretion to determine whether the guilt of the accused is strong. Judicial discretion is
the domain of the judge before whom the petition for provisional liberty will be decided. The
mandated duty to exercise discretion has never been reposed upon the prosecutor.”

Imposed in Baylon v. Sison was this mandatory duty to conduct a hearing despite the
prosecution’s refusal to adduce evidence in opposition to the application to grant and fix bail.
We quote below the pertinent portion of the Decision therein:

“The importance of a hearing has been emphasized in not a few cases wherein the Court ruled
that even if the prosecution refuses to adduce evidence or fails to interpose an objection to the
motion for bail, it is still mandatory for the court to conduct a hearing or ask searching questions
from which it may infer the strength of the evidence of guilt, or the lack of it, against the
accused.” In Gimeno v. Arcueno, Sr., the Court also held: “The grant of bail is a matter of right
except in cases involving capital offenses when the matter is left to the sound discretion of the
court. That discretion lies, not in the determination whether or not a hearing should be held but
in the appreciation and evaluation of the prosecution’s evidence of guilt against the accused. x x
x A hearing is plainly indispensable before a judge can aptly be said to be in a position to
determine whether the evidence for the prosecution is weak or strong.”
Basco v. Rapatalo summarized several cases that emphasized the mandatory character of a
hearing in a petition for bail in a capital case. It enunciated the following duties of the trial judge
in such petition:

“(1) Notify the prosecutor of the hearing of the application for bail or require him to submit his
recommendation (Section 18, Rule 114 of the Rules of Court as amended);

“(2) Conduct a hearing of the application for bail regardless of whether or not the prosecution
refuses to present evidence to show that the guilt of the accused is strong for the purpose of
enabling the court to exercise its sound discretion (Sections 7 and 8, supra);

“(3) Decide whether the evidence of guilt of the accused is strong based on the summary of
evidence of the prosecution (Baylon v. Sison, supra);

“(4) If the guilt of the accused is not strong, discharge the accused upon the approval of the
bailbond. (Section 19, supra). Otherwise, petition should be denied.

The Court added: “The above-enumerated procedure should now leave, no room for doubt as to
the duties of the trial judge in cases of bail applications. So basic and fundamental is it to
conduct a hearing in connection with the grant of bail in the proper cases that it would amount to
judicial apostasy for any member of the judiciary to disclaim knowledge or awareness thereof.”
Additionally, the court’s grant or refusal of bail must contain a summary of the evidence for the
prosecution, on the basis of which should be formulated the judge’s own conclusion on whether
such evidence is strong enough to indicate the guilt of the accused. The summary thereof is
considered an aspect of procedural due process for both the prosecution and the defense; its
absence will invalidate the grant or the denial of the application for bail.

Clearly, the grant of bail by Executive Judge Santiago was laced with grave abuse of discretion
and the Court of Appeals was correct in reversing him.
OKABE VS GUTIERREZ

2004, CALLEJO, SR. J.:

DOCTRINE: Section 26, Rule 114 of the Revised Rules on Criminal Procedure, viz.: SEC. 26.
Bail not a bar to objections on illegal arrest, lack of or irregular preliminary investigation.—An
application for or admission to bail shall not bar the accused from challenging the validity of his
arrest or the legality of the warrant issued therefor, or from assailing the regularity or
questioning the absence of a preliminary investigation of the charge against him, provided that
he raises them before entering his plea. The court shall resolve the matter as early as
practicable but not later than the start of the trial of the case. It bears stressing that Section 26,
Rule 114 of the Revised Rules on Criminal Procedure is a new one, intended to modify previous
rulings of this Court that an application for bail or the admission to bail by the accused shall be
considered as a waiver of his right to assail the warrant issued for his arrest on the legalities or
irregularities thereon. The new rule has reverted to the ruling of this Court in People v. Red. The
new rule is curative in nature because precisely, it was designed to supply defects and curb
evils in procedural rules. Hence, the rules governing curative statutes are applicable. Curative
statutes are by their essence retroactive in application. Besides, procedural rules as a general
rule operate retroactively, even without express provisions to that effect, to cases pending at the
time of their effectivity, in other words to actions yet undetermined at the time of their effectivity.
Before the appellate court rendered its decision on January 31, 2001, the Revised Rules on
Criminal Procedure was already in effect. It behooved the appellate court to have applied the
same in resolving the petitioner’s petition for certiorari and her motion for partial reconsideration.

FACTS: Maruyama charged Okabe of Estafa. After the preliminary investigation, an Information
was filed and a warrant of arrest was issued.  Petitioner posted a personal bail bond in the said
amount, duly approved by Judge Demetrio B. Macapagal, the Presiding Judge of Branch 79 of
the RTC of Quezon City, who forthwith recalled the said warrant. The approved personal bail
bond of the petitioner was transmitted to the RTC of Pasig City on June 21, 2000. Upon her
request, the petitioner was furnished with a certified copy of the Information, the resolution and
the criminal complaint which formed part of the records of the said case. Petitioner twice left the
Philippines but returned. The prosecution moved for the issuance of a hold departure order to
hold and prevent any attempt on the part of the petitioner to depart from the Philippines.
Petitioner filed a Very Urgent Motion To Lift/Recall Hold Departure Order and/or allow her to
regularly travel to Japan. Petitioner filed a motion for the postponement of her arraignment
alleging that, in case the trial court ruled adversely thereon, she would refuse to enter a plea
and seek relief from the appellate court. The court denied the petitioner’s motions on the ground
that when the petitioner posted a personal bail bond for her provisional liberty, she thereby
waived her right to question the court’s finding of the existence of probable cause for her arrest
and submitted herself to the jurisdiction of the court, more so when she filed the motion for the
lifting of the hold departure order the court issued, and the motion to defer the proceedings and
her arraignment.

ISSUE: Is the application for or filing of bail bond a waiver of one’s right to assail the warrant
issued for his arrest?

HELD: NO. There is no waiver in application for or filing of a bail.

It bears stressing that Section 26, Rule 114 of the Revised Rules on Criminal Procedure is a
new one, intended to modify previous rulings of this Court that an application for bail or the
admission to bail by the accused shall be considered as a waiver of his right to assail the
warrant issued for his arrest on the legalities or irregularities thereon. The new rule has reverted
to the ruling of this Court in People v. Red. The new rule is curative in nature because precisely,
it was designed to supply defects and curb evils in procedural rules. Hence, the rules governing
curative statutes are applicable. Curative statutes are by their essence retroactive in application.
Besides, procedural rules as a general rule operate retroactively, even without express
provisions to that effect, to cases pending at the time of their effectivity, in other words to actions
yet undetermined at the time of their effectivity. Before the appellate court rendered its decision
on January 31, 2001, the Revised Rules on Criminal Procedure was already in effect. It
behooved the appellate court to have applied the same in resolving the petitioner’s petition for
certiorari and her motion for partial reconsideration.

Moreover, considering the conduct of the petitioner after posting her personal bail bond, it
cannot be argued that she waived her right to question the finding of probable cause and to
assail the warrant of arrest issued against her by the respondent judge. There must be clear
and convincing proof that the petitioner had an actual intention to relinquish her right to question
the existence of probable cause. When the only proof of intention rests on what a party does,
his act should be so manifestly consistent with, and indicative of, an intent to voluntarily and
unequivocally relinquish the particular right that no other explanation of his conduct is
possible. In this case, the records show that a warrant was issued by the respondent judge in
Pasay City for the arrest of the petitioner, a resident of Guiguinto, Bulacan. When the petitioner
learned of the issuance of the said warrant, she posted a personal bail bond to avert her arrest
and secure her provisional liberty. Judge Demetrio B. Macapagal of the RTC of Quezon City
approved the bond and issued an order recalling the warrant of arrest against the petitioner.
Thus, the posting of a personal bail bond was a matter of imperative necessity to avert her
incarceration; it should not be deemed as a waiver of her right to assail her arrest.

G.R. No. 180109


July 26, 2010

PEOPLE OF THE PHILIPPINES, Petitioner,


vs.
JOSEPH "JOJO" V. GREY, FRANCIS B. GREY, and COURT OF APPEALS-CEBU CITY,
EIGHTEENTH DIVISION, Respondents.
Doctrine: It is well to remember that there is a distinction between the preliminary inquiry which
determines probable cause for the issuance of a warrant of arrest and the preliminary
investigation proper which ascertains whether the offender should be held for trial or be
released. The determination of probable cause for purposes of issuing the warrant of arrest is
made by the judge. The preliminary investigation proper – whether or not there is reasonable
ground to believe that the accused is guilty of the offense charged – is the function of the
investigating prosecutor.

Facts: On December 11, 2006, an Information for Murder was filed against respondent Joseph
Grey, former Mayor of San Jorge, Samar; his son, respondent Francis Grey; and two others for
the death of Rolando Diocton, an employee of the San Jorge municipal government, before the
Regional Trial Court (RTC), Branch 41, Gandara, Samar. The Information was accompanied by
other supporting documents and a motion for the issuance of a warrant of arrest. RTC Branch
41 Presiding Judge Rosario Bandal denied the motion for the issuance of a warrant of arrest.
Judge Bandal found the prosecution’s evidence to be insufficient to link respondents to the
crime charged. She directed the prosecution to present, within five days, additional evidence
that would show that accused were the assailants or that they conspired, confederated, or
helped in the commission of the crime charged. Respondents filed a petition for review with the
Secretary of Justice. The Secretary of Justice ruled that the evidence adduced against
respondents was sufficient to establish probable cause for the offense charged. Judge Navidad
proceeded with the preliminary inquiry on the existence of probable cause, and ruled that the
finding of probable cause was supported by the evidence on record. He then issued warrants of
arrest against respondents and all but one of their co-accused. The CA held that Judge Navidad
failed to abide by the constitutional mandate for him to personally determine the existence of
probable cause. According to the CA, nowhere in the assailed Order did Judge Navidad state
his personal assessment of the evidence before him and the personal justification for his finding
of probable cause. It found that the judge extensively quoted from the Joint Resolution of the
Provincial Prosecutor and the Resolution of the Secretary of Justice, and then adopted these to
conclude that there was sufficient evidence to support the finding of probable cause .Hence this
petition for review.

Issue : whether or not Judge Navidad failed to personally determine the existence of probable
cause?

Held :
NO
In this case, the judge, upon his personal examination of the complaint and evidence before
him, determined that there was probable cause to issue the warrants of arrest after the
provincial prosecution, based on the affidavits presented by complainant and her witnesses,
found probable cause to file the criminal Information. This finding of the Provincial Prosecutor
was affirmed by the Secretary of Justice.To establish political harassment, respondents must
prove that the public prosecutor, not just the private complainant, acted in bad faith in
prosecuting the case or has lent himself to a scheme that could have no other purpose than to
place respondents in contempt and disrepute. It must be shown that the complainant possesses
the power and the influence to control the prosecution of cases.Likewise, the allegation that the
filing of the complaint was politically motivated does not serve to justify the nullification of the
informations where the existence of such motive has not been sufficiently established nor
substantial evidence presented in support thereof.
Tamargo vs. Awingan 2010

Doctrine:

The Judge when confronted with a motion to withdraw an Information (on the ground of
lack of probable cause to hold the accused for trial based on a resolution of the DOJ Secretary),
the trial court has the duty to make an independent assessment of the merits of the motion. It
may either agree or disagree with the recommendation of the Secretary. Reliance alone on the
resolution of the Secretary would be an abdication of the trial court’s duty and jurisdiction to
determine a prima facie case. The court must itself be convinced that there is indeed no
sufficient evidence against the accused.

Facts:

In this case, Atty. Franklin V. Tamargo and his 8-year-old daughter were shot and killed
in 2003. The police had no leads on the perpetrators of the crime until a certain Reynaldo Geron
surfaced and executed an affidavit wherein he stated that a certain Lucio Columna told him
during a drinking spree that Atty. Tamargo was ordered killed by Lloyd Antiporda and that he
(Columna) was one of those who killed Atty. Tamargo. Columna was arrested.

On March 8, 2004, Columna executed an affidavit wherein he admitted his participation


as “look out” during the shooting and implicated Romulo Awingan as the gunman and one
Richard Mecate. He also tagged as masterminds Licerio Antiporda, Jr. and his son, Lloyd
Antiporda, ex-mayor and mayor, respectively, of Buguey, Cagayan.

Pursuant to this affidavit, petitioner Harold V. Tamargo (brother of Atty. Tamargo) filed a
complaint against those implicated by Columna in the Office of the City Prosecutor of Manila.
Columna affirmed his affidavit before the investigating prosecutor.

During the preliminary investigation, Licerio presented Columna’s handwritten letter


wherein the latter disowned the contents of his earlier affidavit and narrated how he had been
tortured until he signed the extrajudicial confession. Thus, the investigating prosecutor
recommended the dismissal of the charges.

However, In another handwritten letter addressed to City Prosecutor, Columna said that
he was only forced to withdraw all his statements against respondents during the clarificatory
hearing because of the threats to his life inside the jail.

The RTC judge denied the motion to withdraw the informations and held that based on
the March 8, 2004 affidavit which Columna affirmed before the investigating prosecutor, there
was probable cause to hold the accused for trial.
Aggrieved by the dismissal of the charges, petitioner filed an appeal to the Department
of Justice (DOJ).

However, on August 12, 2005, Secretary Gonzales granted the Antipordas’ motion for
reconsideration (MR) and directed the withdrawal of the Informations. This time, he declared
that the extrajudicial confession of Columna was inadmissible against respondents and that,
even if it was admissible, it was not corroborated by other evidence. As a result, on August 22,
2005, the trial prosecutor filed a motion to withdraw the Informations. On October 4, 2005,
Secretary Gonzalez denied petitioner’s MR.

The RTC, through Judge Cielito Mindaro-Grulla, granted the motion to withdraw the
Informations in an order dated October 26, 2005.

Issue:

Whether the RTC judge gravely abused her discretion because she arbitrarily left out of
her assessment and evaluation the substantial matters that the DOJ Secretary had fully taken
into account in concluding that there was no probable cause against all the accused.

Held:

Yes.

It is settled that, when confronted with a motion to withdraw an Information (on the
ground of lack of probable cause to hold the accused for trial based on a resolution of the DOJ
Secretary), the trial court has the duty to make an independent assessment of the merits of the
motion. It may either agree or disagree with the recommendation of the Secretary. Reliance
alone on the resolution of the Secretary would be an abdication of the trial court’s duty and
jurisdiction to determine a prima facie case. The court must itself be convinced that there is
indeed no sufficient evidence against the accused.

We agree with the CA that Judge Daguna limited herself only to the following: (1)
Columna’s affidavit dated March 8, 2004 wherein he implicated the respondents in the murders;
(2) his affirmation of this affidavit during the April 19, 2004 clarificatory hearing; (3) his letter
dated October 29, 2004 and (4) the May 30, 2005 DOJ resolution upholding the
prosecutor’s recommendation to file the murder charges.28

She completely ignored other relevant pieces of evidence such as: (1) Columna’s May 3,
2004 letter to respondent Lloyd Antiporda narrating the torture he suffered to force him to admit
his participation in the crimes and to implicate the respondents; (2) his May 25, 2004 affidavit
where he stated that neither he nor the respondents had any involvement in the murders and (3)
his testimony during the October 22, 2004 clarificatory hearing wherein he categorically affirmed
his May 3, 2004 letter and May 25, 2004 affidavit.

The selectivity of respondent RTC Judge for purposes of resolving the motion to
withdraw the informations effectively sidetracked the guidelines for an independent assessment
and evaluation of the merits of the case. Respondent RTC Judge thus impaired the substantial
rights of the accused. Instead, she should have made a circumspect evaluation by looking at
everything made available to her at that point of the cases. No less than that was expected and
required of her as a judicial officer.
Executive Summary

Question:

This Petition for Review is the culmination of a wrong-headed approach by a bonding company
to acquit itself of liability on purportedly spurious bail bonds issued in its name. Even if we
concede the basic premise the questioned bail bonds are indeed false, there are prescribed
remedies under our procedural rules which the surety simply failed to avail of despite ample
opportunity. Hence, although the lower court decisions under review are not free of flaws the
Court is impelled to deny the petition.

Is RTC been sufficiently convinced that the questioned bail bonds were indeed spurious.?

Answer: NO. There are grounds entrenched in jurisprudence for the quashal of a writ of
execution, yet such quashal rests largely in the discretion of the court, that will be exercised in
the furtherance of justice. In this case, had the RTC been sufficiently convinced that the
questioned bail bonds were indeed spurious, there would be grounds in equity for the writs of
execution to be set aside. After all, the notion that an entity can be held liable for an obligation it
did not actually contract offends basic principles of justice.

However, the RTC was not sufficiently convinced, preferring instead to await definitive word
from the Insurance Commission on the revocation of Reliance’s former agent. One can view the
justification as a measure of prudence, or disagree with it as an abdication of the judicial duty to
decide. Yet concededly, the RTC’s discretion in deciding the matter is entitled to great respect,
not only due to the fact that the matter for consideration is the quashal of writs of execution, but
also because the trial court is normally deemed as the most capable trier of facts under the
circumstances.

Yet ultimately, this case does not pivot on whether the RTC correctly refused to set aside the
writs of execution. At this stage, despite the numerous errors of procedure already committed by
Reliance, there still was leeway for the allowance of its prayer for discharge, since the quashal
of writs of execution was obtainable as a remedy against issuances of inequitable nature.
However, Reliance instead again committed another fundamental procedural error, one that
whisks away whatever sympathy it may have acquired owing to its basic position.

Simply put, appeal does not lie as the remedy from an order denying a motion to set aside a writ
of execution. Appeal avails as a remedy only against judgments or final orders, a general rule
that holds true whether for civil or criminal procedure. Appeal may have been properly available
from the five judgments on the bail bonds in the five criminal cases, as such judgments would
have constituted as the final orders on the matter whether Reliance should be held liable on
these bonds. However, appeal cannot be undertaken from the RTC’s Order, arising as it did, at
the execution stage.

You might also like